Sunteți pe pagina 1din 106

Compete Math: Hard Math Taught 

Right 

AMC 10 Curriculum 
 

Quinn Perian 

1
Table of Contents 
 
AMC 10 Curriculum  
Introduction 3 
1 Algebra 4 
1.1 Statistics 5 
1.2 Vieta’s Formulas 8 
1.3 Sequences and Series 13 
1.4 Equations in Word Problems 17 
2 Combinatorics 22 
2.1 Complementary Counting 23 
2.2 Principle of Inclusion and Exclusion 26 
2.3 Casework 30 
2.4 Pascal’s Triangle 33 
3 Geometry 38 
3.1 Areas 39 
3.2 Circles 44 
3.3 Analytical Geometry 54 
3.4 Three-Dimensional Geometry 60 
3.5 Similar and Congruent Triangles 65 
3.6 Trigonometry 73 
4 Number Theory 82 
4.1 Euclidean Algorithm 83 
4.2 Modular Arithmetic Part I 87 
4.3 Modular Arithmetic Part II 92 
Solutions 96 

2
Introduction 
 

First of all, the content of this book is also available at the web address
competemath.weebly.com. At that website, you can also submit your solutions to exercises in
order to receive feedback.

This book is intended to help you prepare for math competitions. I believe that students learn
best when they are taught to understand how and why things work. I also believe that the best
way to cement a deep understanding of concepts into your mind is by working on problems. The
problems in this book are challenging and are meant to help students learn to solve problems as
well as learning the concepts. Thus, each concept taught will start with sample problems to
introduce it, as well as problems at the end for further practice. In addition, for any student using
this book, I highly recommend that you attempt all of the sample problems on your own before
reading the given solution; you'll gain a much deeper understanding of the topics, and your
problem solving skills will become much better (even if you don't solve all of the sample
problems)! Enjoy!

3
1 Algebra 
1.1 Statistics 5 
1.2 Vieta’s Formulas 8 
1.3 Sequences and Series 13 
1.4 Equations in Word Problems 17 
 
 

4
1.1 Statistics  
Sample  Problem  1.  Find  the  average  (also  called  mean)  of 
,  ,  ,  and  .  How  can  we 
use this result? 

We  could  just  compute  all  of  the  sums  and  take  the  average  of  those  numbers,  but  we 
will  leave  the  numbers  in  the  forms  they  are  above  to  try  to  see  if  we  can  find  any 
interesting  patterns  that  we  could  generalize.  Observe  that 
.  We  notice  that  because 
(a-3)+(b-3)+(c-3)+(d-3)=0, 3 ends up being the average. Let's try to generalize this. 

Sample  Problem  2.  Find  the  average  (also called mean) of  , 


,  ,...,  and    where  . 
How can we use this result? 

First  of  all,  we  note  that  there  are    terms.  Thus,  we  can  note  that  our  average  is 

.  From  this  result,  we  get  a  few  useful  realizations. 


First  of  all,  note  that  if  we  let    be  the  average  of  the  terms  in  the  series,  then  we  get 
.  By  thinking  further  about  what  this  means,  we can see that the 
sum  of  the  average  minus  each  term  in  the  sequence  is  equal  to  . Also, note that if we 
are  trying  to  quickly  find  the  average  of  a  sequence,  we can guess a number, say n, and 
find  the  average  of  s-n for each s that is a term of our sequence. The average of all terms 
in  the  sequence  is  then  just  n  plus  the  average  of  all   (this  is  simply  restating  the 
formula we just derived). 

Sample  Problem  3.  If  the  median  of  a set of 5 positive integers is 8 


and  their  average  is  9,  what  is  the  largest  possible  integer  that 
could be in the set? 

To  make  this  problem  easier  to  think  about,  we  will  write  out  the  set  of  numbers  in 
non-decreasing  order  so  it  looks  like  this:  _  _  8  _  _.  Next,  using  what  we  got  in  the  last 

5
problem,  note  that  another  way  to  say that we want to get the largest possible integer is 
to  say  that  we  want  the  integer  that  is  greater  than  9  by  the  largest  possible  margin. 
Note  that  9  minus  each  term  in  our  sequence  adds  to  0,  so  we  want  all  terms  but  the 
largest  term  to  be  as  small  as  possible.  Other  than  the  8  term  and  the  largest  term,  we 
need  1  term  greater  than  or  equal  to  8  and  two  terms  less  than  or  equal  to  8.  Thus,  the 
minimum  values  of  the  terms other than the largest one would be 1, 1, 8, and 8. We note 
that  the  sum  of  each  of  these  integers  minus  9  is  -1-1-8-8=-18,  so  our  largest  possible 
positive integer is 9+18 =27. 

Sample  Problem  4.  If  group  A  had  an  average  test  score  of  90, 
group  B  had  an  average  test  score  of  80, and together they had an 
average  test  score  of  83,  find  the  ratio  of  the  number  of  people  in 
group A to the number of people in group B.  

Using  sample  problem  1,  we  note  that  the  total  decreases  from  83%  equals  the  total 
increases.  Thus,  we  will  let  the  scores  of  the  people  in  group  A  be  ,  , 
,  .  .  .,  and  .  Similarly,  we  will  let  the  scores  of  the  people in  group B be 
,  ,  ,  .  .  .,  and  .  Thus, 
.  However,  using  sample  problem  1, 
we  know  that    because  80-83=-3.  Similarly,  we 
know  that  .  Thus,  ,  so 
the  ratio  of  people  in  group  A  to  group  B  is  3  to  7.  Try  to  think  about  this  solution  to 
generalize this strategy. 

6
Exercises 

1.1.1)  If  the  median  of  set  S  is  8,  the  average  is  ,  it  has  n  elements,  the  range is 1, and 
all  of  its  elements  are  integers,  find  n  (there  could  be  multiple possible values of n; find 
them all)? 

1.1.2)  Whenever  an  element  with  value  v  is  removed  from  a  set  with  n  elements  and 
average a, what does the new average become? 

 
 

7
1.2 Vieta’s Formulas 
Sample  Problem  1.  Find  the  sum  of  the  roots,  not  necessarily 
distinct, of the quadratic  

When  looking  at  this  problem,  the  first  solution  that  comes  to  mind  is  just  to  find  the 
roots  and  to  add  them  up.  By  the  quadratic  formula,  we  see  that  the  roots  of  the 

polynomial  are   Adding the roots together, we find their sum to be  . After 


noticing  that  this  answer  is  merely    times the coefficient of the middle term, we may 
wonder  if  this  is  always  true.  Indeed,  it is (at least it is if the coefficient of the   term is 
1), and it is relatively easy to prove.  

Sample  Problem  2.  Find  the  sum  of  the  roots,  not  necessarily 
distinct, of the quadratic  

Using  the  same  method  as  we  did  in  the  previous  problem,  we  get  that  the  roots  are 

  Next,  note  that 

  We  may 
wonder if this can be extended to the product of the roots. 

Sample  Problem  3.  Find  the  product  of  the  roots,  not  necessarily 
distinct, of the quadratic  

Just  as  before,  we  know  that  the  roots  of  the  quadratic  are    Next,  we 
multiply  and  use  the  difference  of  squares  to  find  that 

 Now that both of these expressions 
turned  out  nicely,  we  wonder  if  there  is  a nicer explanation that could generalize this to 
higher degree polynomials. 

8
Sample  Problem  4.  Relate  the  sum  of  the  roots  and  the  coefficient 
of the term in (x+1)(x+2)(x+3). 

To  start,  we  will  think  about  how  to  expand  this  in  a  way that may be slightly different 
than  the  way  you  are  used  to,  in  order  to  not  expand  the  whole  thing.  When  we're 
expanding  a  bunch  of  binomials  multiplied  by  each  other,  the  result will be the sum 
of  each  possible  combination  of  choosing  one  of  the  two  terms  in  each  of  the 
binomials  and  multiplying  the  chosen  terms  together.  For  example,  in  this  situation, 
we  can  choose  either  the  x  term  or  the  constant  term  from  each  binomial  to  multiply 
into  a  factor.  Because  we  want  the    term,  we  need  to  choose  the  x  term  from  two  of 
the  binomials  and  the  constant  term  from  the  other.  We  can  do  this  in  three  ways, 
depending  on  which  binomial  we choose a constant from. Thus, the only relevant terms 
we  get  from  expanding  this  are  ,  ,  and  .  Thus,  we  get  the 
coefficient  of  the    term  to  be  1+2+3.  We  note  that  this  coefficient  is  just  -1  times  the 
sum of the roots.  

Let's  look  at  expanding  a  quadratic  using  the  bolded  method  to  make  sure  you 
understand  it.  We  will  expand  (x+1)(x+2).  We  can  choose  either  a  1  or  an  x  to  multiply 
in  from  the  first  binomial  and  a  2  or  an  x  from  the  second.  Thus,  we  get  the  possible 
combinations  of  choosing  one  term  from  each  binomial  to  be  (x)(x),  (x)(2),  (1)(x),  and 
(1)(2).  Adding  these  together  we  get  the expanded term to be  . If you don't 
understand why this method works, think about starting with the binomial (a+b). When 
we  multiply  (a+b)  by  (c+d),  we  get  (a+b)(c+d)=c(a+b)+d(a+b)=ca+cb+da+db.  In  other 
words,  we  can  multiply  each  term  of  our  first  binomial  by  either  of  the  terms  in  the 
polynomial we multiplied it by, and then we add up each case. 

Sample  Problem  5.  Find  the  sum  of  the  roots,  not  necessarily 
distinct,  of  the  polynomial 
 

This  time,  we  can't  just  solve  for  the  roots  and  add  them,  so  we  will  need  to  be  clever. 
When  stuck  on  a  problem,  it  is  often  helpful  try  to  represent  the  given information in a 
different  way.  Because  we  are  dealing  with  the  roots  of a polynomial, we think of using 
the  Fundamental  Theorem  of  Algebra.  By  the  Fundamental  Theorem  of  Algebra  (look 
this  up  if  you want more detail, but the proof of this theorem is beyond the scope of this 

9
curriculum),  we  have  a  unique  factorization  of  our  polynomial 
  where  each    is  a  root  of  . 

Looking  at  problems  2  and  4,  we  think  we  might  get   
because,  in  both  problems,  we  found  the  sum  of  the  roots  to  be  -1  times  the  second 
rightmost  coefficient  divided  by  the  first  coefficient.  We  can  note  that  the    is  the 
coefficient  of  the   term in the first representation of our polynomial. Thus, we want 
to  find  the  coefficient  of  the    term  in  the  second  representation  of  our polynomial. 
By  thinking  about  what  the  polynomial  would  look  like  if  we  expanded  it  and  only 
paying  attention  to  terms  in  the  form  of    for  some  constant  c,  we  can see that the 
coefficient  is  equal  to  .  This  is because, of the k factors of   
that  can  contribute  an    or  a  ,  exactly    must  contribute  an  x  and  the  other 
must  contribute a   for some   in order to produce one of the terms we are looking at. 
Therefore,  we  see  that    and  thus, 

  Seeing  two  coefficients  represent  operations  done  on 


the roots, we are curious if we can find more generalizations.  

Sample  Problem  6.  Generalize  what  we  found  in  sample  problem 
4 to the other coefficients of higher degree polynomials. 

By  looking  at  our  result  in  problem  3,  we  can  apply  similar  methods  by  looking  at  the 
  term  (think about how many non-x terms need to multiply together to form one of 
those  terms)  for  .  By  using  our  alternate  method  of  expansion  to  express  each 
coefficient in terms of the roots of the polynomial, we get the following equations. 

. . . 

10
To  be  clear,  we  start  with  the  sum  of  the  roots,  then  go  to  the  sum  of  every  possible 
product  of 2 roots, then the sum of every possible product of 3 roots, and so on, until we 
get  to  the  sum  of  every possible product of k roots. These are formulas are called Vieta's 
Formulas, and they can be used for a wide variety of problems. 

11
Exercises 
1.2.1)  For  positive  integer  ,  the    roots  (real  and  complex)  of    are  called  the   
roots  of  unity.  For  example,  the    roots  of  unity  are  ,  ,  ,  and  . Given that there 
are    distinct    roots  of  unity  (this  can  be  proved  relatively  easily  using  the complex 
number  plane;  you  can  find  more  extensive  information  on  this  with  a  quick  google 
search), find the sum of the roots of unity for . 

1.2.2)  Find  the  product  of  all  possible  q  if  a,  b,  and  c  are  the  roots  of 
  and 

 

 
 
 

12
1.3 Sequences and Series 
Before  we  get  into  this  lesson,  you  should  already  know  the  formula  for  the  sum  of  an 
arithmetic  series and how to prove it. If not, there are many good resources online about 
this formula that you can find to learn about it. 

Sample  Problem  1.  Find  the  sum  of    in 


terms  of  n  (give  a  closed  form  for  the  expression  with  a  constant 
number of terms). 

There  is  no  clear  solution  at  first,  so  we  start  by  thinking  about  what  aspects  of  this 
problem  make  it  hard.  The  main  challenge  in  this  problem  is  that  we  don't  know  the 
number  of  terms.  The  other  challenge  in  this  problem  is  that  the  terms  change  by 
multiplication,  but  we  are  trying  to  find  the  sum  of  the  terms.  In  order  to  reduce  the 
number  of  terms  to  a  constant  number,  we  think  about  subtracting  the  sequence  from 
itself  somehow.  We  let  .  Evidently,  just  subtracting  the 
sequence  from  itself  won't  help,  so  we think about how we could do something slightly 
different.  Thinking  about  how  each  term  is just 3 times the previous term, we think that 
maybe  we  should  multiply  the  sequence  by  something  and  then  subtract  the  product 
from  the  sequence.  It  seems  natural  to  multiply  the  sequence  by 3 because it essentially 
just  makes  the  sequence  start  at   and end at  , while still containing the unknown 
number  of  terms  in  the  middle.  Thus,  we  write  . 
Because  this  sequence  is  so  similar  to  ,  we  see  that 
subtracting  them  would  get  a  lot  of  terms  to  cancel  out.  Thus,  we  have 

Finally, we see that we have . 

We  can  generalize  this  example  very  easily  by  using  the  above  process  to  see  that 

.  Note  that  this  does  work  with  the  above 


example  because 
.  The  process 
we  used  of  subtract  the  sequence  from  a  multiple  of  itself may seem a little bit odd, but 
it  does  indeed  work  perfectly  fine.  Note  that  the  infinite  geometric  series 

13
,  going  on  forever,  is  equal  to    for  r<1, 
because,  for  r<1,    goes  to  zero  as  n  goes  to  infinity  (this  is  far  from  a  rigorous  proof, 
but  it  should  give  an  intuitive  reason  because  any  fully  rigorous  proof  would  be  far 
above the level of this curriculum). 

Sample  Problem  2.  Find  ,  with 


infinite square roots. 

For  this  problem,  we  won't  be  fully rigorous with the idea of infinity, but you should be 


able  to  get  an  intuitive  understand  of  why  this  solution  works.  Similar  to  what  we  did 

for  the  last  problem,  we  will  let  .  However,  because 


there  are  infinitely  many  square  roots,  there  are  still  infinitely  many square roots inside 

the  first  one.  In  other  words,  we  see  that   


(again,  we  are  not  being  fully  rigorous  with  the  idea  of  infinity  here).  Thus,  we  simply 
square  both  sides  of    to  get  .  We  then  simply  solve  the 
quadratic  to  get  S=2  or  S=-1.  However,  the  value  of  S  is  clearly  positive,  so  -1  is  an 
extraneous solution, so we have that S=2, and we are done. 

Sample  Problem  3.  Find  the  value  of 

This  problem,  again,  is  hard  because  the  number  of  terms isn't constant. Further, in this 
problem,  there  doesn't  appear  to  be  an  easy  way  to  subtract  it  from  itself  as  we  did  in 
the  first problem. However, we recall that the key to the first problem was a ton of terms 
cancelling.  Thus,  we  think  of  trying  to  rewrite  this  sequence  to  make  a  bunch  of  terms 
cancel.  Because  each  pair  of  consecutive  terms  has  a  number  in  the  denominator  in 
common  with  the  denominator  of  the  second  term,  we  think  of  using  that.  Thus,  we 

guess  that   because because it would make all of the terms cancel out 


nicely.  As  you  can  easily  verify,  this  equality  does  indeed  hold  true.  Therefore,  we  can 

14
rewrite  this  sum  as  .  This 
method  of  rewriting  the  terms  of  the  sequence  in  such  a  way  to  make  almost  all  of  the 
terms  cancel  out  and  leave  us  with  a  simple  to evaluate expression is called telescoping 
because  the  series  collapses  like  a  folding  telescope  does.  Telescoping  can  be  done 
without rewriting the terms through cleverly multiplying by a term or other methods. 

15
Exercises 

1.3.1) Prove that using telescoping. 

 
 
 
 
 
 
 
 
 

16
1.4 Equations in Word Problems 
Rate Problems 

Sample  Problem  1.  Car  A  travels  at  a  speed  of  25  mph.  If  car  B 
starts  5  miles  behind  car  A,  at  what  speed  does  car  B  travel  if  it 
catches up with car A in 30 minutes? 

For  this problem, we can use the equation d=rt, where d is the distance, r is the rate, and 
t  is  the  time.  If  this  equation  doesn't  appear  to  be  obviously  true  immediately,  think 
about  how  the  units  will  cancel  out.  We  will  want  to  introduce  some  variables  to 
represent  what  we  already  know  and  what  we  want  to  know.  In  general,  when  doing 
word  problems,  it  is  often  a  good  idea  to  start  by  defining  variables.  Let  ,  , and   
be  the  distance  car  A  travels,  the rate at which car A travels, and the amount of time car 
A  travels.  We  will  define  ,  ,  and    similarly.  Next,  we  will  look  at the information 
we  are  given.  We  know that  . We are also given that when  , 
.  Thus,  we  know  that  .  By  writing  the  equation 
,  we  can  see  that  . Another way to look at this 
problem,  that  is  essentially  the  same,  but  a  little  quicker,  is  to  see  that  car  B  travels  5 
more  miles  than  car  A  in  half  an  hour,  so  car B has a speed   greater than car A's 
speed.  Thus,  car  B  has  a  speed  of  ,  as  we  saw  before.  One  important  thing  to 
note,  and  to  be  careful  of,  is  that we are using mph as speed, so   is only true if all 
of  the  units  are  the  same,  meaning  we  must  note  that  we  used 

17
Sample  Problem  2.  Fred  runs  at  a  speed  of  7  mph.  Ned  runs  at  a 
speed  of  6  mph.  If  Fred  and  Ned  both  start  at the same place on a 
169  mile  track,  how  long  until  Fred  and  Ned  meet  for  the  first 
time once they start running.  

We  will,  again,  use  d=rt.  For  Fred  and  Ned  to meet, they will have had to travel the full 
distance  of  the  track  when  the  distances  each  ran  is  combined.  In  one  hour,  we  can  see 
they  travel  a  total  of    closer  to  each  other.  Thus,  we  see  that  169=13t, 
so they will meet each other for the first time in exactly 13 hours. 

Sample  Problem  3.  If  Fred  biked  up  a  hill  at    mph  and  biked 
down  the  same  hill  at    mph,  find  Fred's  average  speed  across 
the whole ride in terms of and . 

When  we  first  see  this  problem,  we  may  think  that  the  answer  is  just  .  However, 
we  can  intuitively  see  that  his  is  wrong  because Fred is biking for longer when he bikes 
at  a  slower  rate.  For  this  problem,  we  already  have  two variables defined for us and, as 
we  have  seen  previously,  it  is  a  good  idea  to  assign  variables  to  everything.  Thus,  we 
will  call  the  distance  from  the  bottom  to  the  top  of  the  hill    miles.  We  can  then  easily 
see  that  Fred  traveled    miles  in  total.  We  want  to  find  Fred's average speed, which is 
imply  total  distance  divided by total time. We have the distance, so now we just want to 
find  the  time.  For  the  first  part  of  his  ride,  Fred  traveled    miles  at  a  rate  of    mph. 

Thus,  for  the  first  part  he  took    hours.  Similarly,  for  going  down  the  hill,  it took Fred 

  hours.  Thus,  Fred's  total  time  was  +   =    hours.  Finally,  we  can  easily 
find  Fred's  average  rate  by  taking  the  distance  divided  by  the  time  to  get  his  rate  to  be 

This quantity is called the harmonic mean of and . 

18
Work Problems 

Sample  Problem  4.  If  7  workers  can  paint  7  houses in 7 hour, how 


long  would  it  take  for  3  workers  to  paint  3  houses?  Assume  each 
worker  works  at  the  same  rate  and  and  each  house  needs  the 
same amount painted. 

Similar  to  our  previous  two  problems,  we  will  use  the  equation  w=rt  where  w  is  the 
amount  of  work,  r  is  the  rate, and t is the time. In this problem, we let the units for w be 
houses  painted,  units  for  r  be  in  houses  per  hour  for  one  worker,  and  t  be  in  hours. 
Thus,  ,  so  each  worker  can  paint   of a house per hour. Thus, 3 workers can a 

paint    of  a  house  in  an  hour.  Thus,  to  paint  3  house,  it  takes  3  workers    hours. 
Before  moving  on,  it is important to note that it took 7 not 3 hours slightly unintuitively. 
We  can  notice  that  this  task  took  the  same  amount  of  time  as  it  did  for  7  workers  to 
paint  7  houses,  so  we  wonder  if  there  is  a  nicer  explanation  for  this.  Indeed,  there  is. 
Using  the  equation  w=rt,  and  solving  for  each  variable,  it  can  be  easily  seen  that  when 
we  change  the  number  of  workers  from a to b, the amount of time gets multiplied by  . 
Similarly,  when  we  change  the  number  of  houses  needing  painting  from  c  to  d,  we 
multiply  the  time  by  .  Thus,  in  the  situation  of  the  problem,  we  are  multiplying  7 
hours  by  .  Make  sure  you  understand  what  happens  when  you  change  the 
number  of  workers  and  the  amount  of  work,  it  can  be  very  easily  generalized  to  other 
problems and will often pop up. 

19
Ratio Problems 

Sample  Problem  5.  In  a  town  of  98  people,    of  the  men  and    of 
the  women  are  married.  Assuming  each  marriage  consists  of  one 
man  and  one  woman  and  consists  of  only  people  in  the  town, 
how many men are in the town. 

Again,  we  will  start  by  defining  variables. Let w be the number of women and m be the 


number  of  men  in  the  town.  Thus,  ,  or  equivalently,    of  the 

total  population.  Thus,  there  are    men.  Just like in many AMC 10 level word 


problems, once we define variables the rest of the steps are fairly intuitive and obvious. 

20
Exercises 
1.4.1)  Albert  is  running  a  lemonade  stand.  He  got  2 more cups than than he had ounces 
of  lemonade.  The  cooler  he  got  could  unfortunately  only  hold  one  less  than  half  of  the 
number  of  cups  he  got.  Finally,  the  cooler  he  had  could hold one more than one fourth 
of the number of ounces of lemonade he. How many ounces of lemonade did he have? 

1.4.2)  Mary  is  running  exactly  one  time  around a track. She ran the first half of the track 

 
in  only  3  minutes.  She  was  then  tired  and  decreased  her  speed  by  1  mph  for the rest of 
distance to have a total time of 7 minutes. How long is the track?

 
 
 
 
 
 

21
2 Combinatorics 
2.1 Complementary Counting 23 
2.2 Principle of Inclusion and Exclusion 26 
2.3 Casework 30 
2.4 Pascal’s Triangle 33 
 

 
 
 
 
 
 

22
2.1 Complementary Counting 
Sample  Problem  1.  How  many  positive  integers  less  than  101  are 
not multiples of 3? 

If  we  start  out  trying  to  to  directly  count  this quantity, it quickly becomes apparent that 
this  is  fairly  hard.  We  find  that  it  is hard to identify which numbers are not multiples of 
3  without  initially  thinking  about  which  numbers  are.  Thus,  it would make sense to try 
to think about the problem in a different way. 

Instead  of  trying  to  count  the number of numbers that are not a multiple of three, it will 


be  much  simpler  to  count  how  many  multiples  of  3  are  below  101  and  subtract  that 
from  the  number  of  total  positive  integers  below 101. We can easily see that there are 33 
positive  multiples  of  3  below  101  and  that  there  a  total  of  100  positive  integers.  Thus, 
there  are  a  total  of  100-33=67  positive  integers  that  are  not  multiples  of  3.  The  strategy 
used  in  this  problem  of  counting  the opposite of what you want and subtracting it from 
the  total is a very potent one. This strategy is called complementary counting.  It is often 
good  to  use  complementary  counting  when  counting  something  directly  seems  hard, 
especially  if  the  problem  asks  explicitly  for  the  number  of  one  thing  that  is  not 
something else or without a certain attribute. 

Sample  Problem  2.  How  many  positive  integers  below  100  are 
multiples of 3 but not 8? 

In  this  problem  it  isn't  apparent  which  numbers  are  multiples  of  3  but  not  multiples  of 
8.  Thus,  we  think  of  using  complementary  counting.  It  is  important  to  be  careful  here, 
because  it  may  at  first  appear  that,  when we count the opposite, we count the multiples 
of  8  but  not  3.  However,  we  instead  need  to  count  the  number  of  positive  integers  less 
than  100  that  are  not  multiples  of  3  or  are  multiples  of  3  and  8.  Thus,  it  is  important  to 
be  careful  in  making  sure  we  count  all  numbers  not  in  our  desired  set  but  in  our  total 
and  to  make  sure  that  we  properly  determine  what  opposite  to  count.  Using 
complementary  counting to solve the problem in this way is perfectly legitimate, but we 
will show another way here for the purpose of introducing a useful concept. 

23
Instead  of  trying  to  use  positive  integers  below  100 as the total, we will instead think of 
a  different  total.  We  will  use  the  multiples  of  3  below  100,  because  then  we  essentially 
reduce  our problem to the previous one. There are 33 multiples of 3 below 100. Next, we 
need  to  remove  the numbers of multiples of 3 that are also a multiple of 8. We can easily 
see  that  each  such  number  will  be  a  multiple  of  24.  We can easily count that there are 4 
positive  multiples  of 24 below 100. Thus, there are 33-4=29 positive integers meeting the 
desired conditions. 

24
Exercises 
2.1.1)  How  many  ways  can  5  beads,  2  blue  and  3 red, be arranged in a line without the 
blue beads being next to each other (beads of the same color are indistinguishable)? 

 
 
 

25
2.2  Principle  of  Inclusion  and 
Exclusion  
Sample  Problem  1.  In  a  school,  there  are  20  student  who  play 
sports,  15  who  play  musical  instruments,  and  5  who  play  a  sport 
and  a  musical  instrument.  How  many  students  play  a  sport  or  a 
musical instrument (or both)? 

When  doing  this  problem,  we  need  to  be  careful  to  not  over  count  any  student.  For 
example,  answers  of  15+20=35  or  15+20+5=40  would  both  be  wrong  because  they  over 
count  the  number  of  students  doing  both.  If  you  are  confused  in  questions  like  these, 
especially  when  they  get  more  complicated,  it  can  often  be  a  good  idea  to draw a Venn 
Diagram like the one seen below. 

By  drawing  a  Venn  Diagram,  we  can  make  sure  to  not  over count any person. Once we 
have  drawn  the  Venn  Diagram,  as  in  the  picture  above,  we  can  often  easily  find  the 

26
desired  value.  In  this  case,  it  is  the  total  number  of  students,  so  we  just  add  up  the 
numbers in the diagram to get a total of 30 students. 

Drawing  a  Venn  Diagram  certainly  works  and  is  a  good  technique  to  remember  when 
you're  confused,  but  it  also  isn't  the  most  efficient.  We  would  prefer  if  there  were  an 
algebraic  way  to  solve  this  problem  without  drawing  out  a  diagram.  Thus,  we  will  try 
to  look  at  exactly  what  we  did  in  drawing  the  Venn  Diagram  in  order  to  find  an 
algebraic  representation  of  our  steps.  In  drawing  this  diagram,  we  were  initially  given 
20  students  who  play  sports  and  15  students  who  play  a  musical  instrument.  We 
subtracted  the  5  people  doing  both  activities  from  the  20  people  playing sports and the 
15  people  playing  an  instrument  in  order  to  find  the  people  doing  just  one  activity.  We 
then  added  the  5  people  back  in  to  account  for  the  people  doing  both  activities.  Thus, 
we subtracted 5 twice and added it once, for a total of subtracting it once.  

The  process  described  above  can  be  easily  generalized  to  see  that  the  total  number  of 
elements  that  are in either one set or another (or both) is equal to the sum of the number 
of elements in each set individually minus the number of elements that are in both sets. 

Sample  Problem  2.  Generalize  the  last  problem  in  order  to  find  a 
general  formula  for  the  total  number  of  elements  in  any  of  n  sets. 
Make  sure  to  not  over  count  any elements. You are given the total 
number  of  elements  in  the intersection of any z sets for each z less 
than or equal to n. 

It  isn't  immediately  apparent  where  to  start,  so  we  will  begin  by  looking  at  another 
example.  We  will  look  at  three  different  sets  and  try  to  find  a  general  formula  for  that 
case.  First of all, if we just add together the number of elements in each set, we will have 
counted  the  elements  in  the  intersection  between  exactly  two  sets  twice  (once  for  each 
set  they  are  in).  Thus,  we  need  to  subtract  the  number  of  elements  in  each  pair  of  two 
different  sets.  Next,  for  the  elements  in  all  3  sets,  we  counted  them  3  times  when  we 

added  the  elements  in  each individual set, and we subtracted them   times when 


we  subtracted  each  element  in  two  sets  (once  for each combination of two sets from the 
3 sets the element is in). Thus, we must add in the number of elements in 3 sets.  

27
From  the  two  previous  examples,  we  may  start  to  see  a  pattern  emerge.  It  would 
appear  that  we  start  by  adding  in  the  number  of  elements  in  each  set,  then  subtract 
the  number  of  elements  in  each  pair  of  sets,  add  the  number  of  elements  in  each 
triplet  of  sets,  subtract  the  number  of  elements  in  each  group  4  sets,  and  so  on, 
alternating adding and subtracting until we get to the elements in all n sets. 

We  will  prove  that  the  above  method  works  by  proving  that  any  arbitrary  element  in 
exactly    sets  will  be  counted  exactly  once,  similar  to  how  we  worked  out  the  total 
number  of  elements  in  at  least  one  set  for  3  total  sets.  It  can  easily  be  verified  that,  for 
any  element  in  exactly    sets  ,  we  add  or  subtract it   times when we add or subtract 
the elements in each group of   sets. Thus, any element in exactly   sets will be counted 

,  where  the  sign  of  the  last  binomial  coefficient 


depends  on  the  parity  (even  or  odd)  of  .  However,  note  that 

  (this  is  because  of  the 


Binomial  Theorem,  which  we  will  prove  in  the  Pascal's  Triangle  lesson). However, note 

that  this  would  imply  ,  or  that  we 


counted this element exactly once as desired. 

While  this  general  formula  is  good  to  know  and  will  be  useful,  it  is  even  more 
important  to  be  able  to  understand  the  derivation  of  the  formula  so  that  you  would 
know  how  to  find  the  number  of  elements  in  at  least    of  the    total  sets.  Also, 
remember  that  if  you  are  ever  stuck  on  a  problem  of this type, using a Venn Diagram is 
always an option, even if it takes a bit longer. 

28
Exercises 
2.2.1)  At  Math  Problem  High  School,  students  either  play bridge, are on the math team, 
play  chess,  or  are  on  the  science  team.  If    students  play  bridge,  chess,  and  are  on  the 
math  team;    students  play  bridge  and  chess  and  are  on  the  science  team;    students 
play  bridge  and  are  on  the  math  and  science  teams;    students  play  chess  and  are  on 
the  math  and  science  teams;  and    students  play  bridge  and  chess and are on the math 
and science teams, how many students are on at least 3 of the 4 teams? 

2.2.2) How many positive integers below 101 are not multiples of 3 or 8? 

 
 

29
2.3 Casework 
Sample  Problem  1.  How  many  triangles  are  in  the  diagram 
below? 

There  are  no  fancy  techniques  for  this  problem  but  to  tough  it  out  and  count  the 
possibilities.  However,  this  doesn't  mean  we  have  to  just  look  for  triangles,  make  tally 
marks  for  each  one  we  find,  and  hope  that  we  don't  over  count  or  miss  anything. 
Instead,  in this problem and every casework problem, the most important thing you can 
do is to have an organized system through which you can count. 

For  this  problem,  there  are  many  ways  to  organize  your  counting,  none  more  correct 
than another. However, in this solution, we will detail 1 specific method of organization. 

First  of  all,  there  are  5  triangles  that  don't  have  any  triangles  inside  of  them.  These 
triangles  are  ABF,  BGF,  BCG,  DCF,  and  EFD.  We  will  proceed  by  casework  on  which 
triangles  contain  each  of  these  triangles.  First  of  all,  we  can  easily  see  that  ABF  is 

30
contained  in  3 triangles (excluding itself). Next, in order to not over count, we can easily 
see  that  there  are  2  triangles  that  contain  BGF but not ABF (excluding BGF). There are 0 
triangles  that  contain  BCG  but  not  ABF  or  BGF  (excluding  BCG).  There  are  0  triangles 
that  contain  DCF  but  not  ABF,  BGF,  or  BCG  (excluding  DCF).  There  are 0 triangles that 
contain  FED  but  not  ABF,  BGF,  BCG,  or  DCF  (excluding  FED).  Thus, there are a total of 
5+3+2=10 total triangles. 

Notice  how  we  made  it  easier  to  count  the  triangles,  avoided  over  counting  easily,  and 
we  are  certain  that  we  counted  each triangle, because we were organized, neat, and we 
had  a  method.  What  method  we  use  or  how  we  organize  our  casework  isn't  that 
important,  as  long  as  we  are  organized,  neat,  have  a  method.  Also,  note  that  other 
concepts  can  be  used  in  combination  with  casework,  such  as  complementary  counting 
and  the  Principle  of  Inclusion  and  Exclusion,  in  order  to  reduce  the  number  of cases or 
to make the casework easier. 

 
 
 
 
 
 
 
 
 
 
 
 
 
 
 
 
 

31
Exercises 
2.3.1) What is the probability of flipping at least 2 heads if you flip 6 coins? 

2.3.2) How many ways can 12 be made out of 1s, 2s, and 4s? 

 
 
 
 
 
 
 

32
2.4 Pascal’s Triangle  

This  is  Pascal's  Triangle.  Each  number,  other  than  the  1  in the top row, is the sum of the 
2  numbers  above  it  (imagine  that  there  are  0s  surrounding  the  triangle).  We  often 
number  the  rows  starting  with  row  0.  We  also  often  number  the  numbers  in  each  row 
going  from  left  to  right,  with the leftmost number being the 0th number in that row. For 
example,  the  1st  number  in  row  2 would be 2 (the 1 to the left of the 2 is the 0th number 
in  the  row). Try to look for some of the many patterns in Pascal's Triangle. In this lesson, 
we will prove some of these patterns. 

Sample  Problem  1.  Prove  that  the  kth  number  in  the  nth  row  is 

We  will  prove  this  by  explaining another interpretation of the number in each place. We 


will  interpret  each  number  as  the  number  of  ways  to  get  to  the  position  of  the  number 
by  only  moving  down  and  left  one  or  down  and  right  one.  We  can  confirm  that  this 
interpretation  works  because  the  number  of  ways  to  get  to  any  position  is  equal  to  the 
sum  of  the  number  of  ways  to  get  to  either  of  the  positions  directly  above  it  (as  these 
two  positions  are  the  only  ones  that  could  lead  to  the  one  below  both  of  them). 
However,  we  can  note  that  to  get  to  the  position  of  the  kth  number  in  the  nth  row,  we 
have  to  go  down  n  times  --  and of those times, exactly k must be down and to the right, 

33
while  the  others  are  down  and  to  the  left  (if  you  can't  see  this  immediately,  imagine 
going  down  and  to  the  left  n  times  to  end  up  on  the  0th  number  in  the  nth row). Thus, 
the  number  of  ways  to  get  to  the  kth  number  in  the  nth  row  is    because  that  is how 
many ways we can choose which k of the n moves down are also to the right. 

The  idea  of  looking  at  each  number  represent  these two different things is a very useful 


one.  Using  this  technique,  we  can  prove  many  useful  identities  about  binomial 
coefficients.  

Sample Problem 2. Find the coefficient of the in . 

It  isn't  immediately  apparent  what  the  coefficients  could  be  or  how  to find them, so we 
will start by looking at some examples. 

34
  

  

  

Does  this  look  familiar?  It  appears  that  the  coefficients    are  exactly  the 
numbers in the nth row of Pascal's Triangle.   

We  now  want  to  figure  out  how  to  prove  this.  As  we  just  saw  in  the  last  problem,  we 
can  rewrite  the  numbers  in  Pascal's  Triangle  as  binomial  coefficients.  We  will  thus 
rewrite the coefficients of our expanded polynomials to see if we notice anything. 

To  start,  we  will  note  that  the  coefficient  of  the    term  is  the  same  as the coefficient of 
the    term.  It  would  also  appear  that  the  coefficient  of  the   term in   is   
Thus, we conjecture that the coefficient of the term in is  

There  is  a  fairly  intuitive  explanation  for  why  this  is  true.  When  we  expand   
we  are  essentially  adding  together  all  the  combinations  of  choosing  an  x  or  a  y  from 
each  of  n  terms  being  multiplied  together.  Thus,  we  are essentially just adding together 

35
each  of  the    ways  to  choose  which  k  of  the  n  terms  contribute  an  x.  This  also 
provides  a  nice  explanation  for  why  the    and    terms  have  the  same  coefficient: 
using  what  we  just  discovered,  the  coefficients  are    and    respectively  (the 
power  of  the  x  and  y  term  must  clearly  always  add  up  to  n  using  the  explanation  we 
gave  earlier  of  how  to  expanding  )  and  =   (if  this  last  statement  isn't 
clear  think  about  how  choosing  k  things  to  include  is  equivalent to choosing n-k things 
not  to  include).  This  statement  about  the  coefficients  of  a  binomial  raised  to  a  positive 
integral power is called the Binomial Theorem. 

Note:  The  Binomial  Theorem  does  in  fact  work  for  a  binomial  raised  to  any  power, but 
the  proof  is  much  too  complicated  to  cover  in  this  curriculum,  and  it  far  beyond  this 
level  of  math.  This  extended  binomial  theorem  also  uses  a  definition  of  binomial 
coefficients  that  is  adapted  to  be  used  with  non-integers  and  negatives.  If  you  want  to 
find  out  more,  a  quick  google  search  for  Extended  Binomial  Theorem  or  Generalized 
Binomial  Theorem  should  do.  This  theorem  can  also  be  generalized  beyond  binomials, 
where  it  is  called  the  Multinomial  Theorem.  A  quick  google  search  for  Multinomial 
Theorem should give information about that. 

36
Exercises 
2.4.1) Prove that the sum of the numbers in the nth row of Pascal's Triangle is . 

2.4.2) Prove that . 

2.4.3) Prove that . 

37
3 Geometry 
3.1 Areas 39 
3.2 Circles 44 
3.3 Analytical Geometry 54 
3.4 Three-Dimensional Geometry 60 
3.5 Similar and Congruent Triangles 65 
3.6 Trigonometry 73 
 

 
 
 
 
 
 
 
 
 

38
3.1 Areas 
Sample  Problem  1.  Find  the  area  of  a  triangle  with  perimeter  p 
whose inscribed circle has a radius of r. 

Just  like  most  geometry  problems,  we  will  start  by  drawing  a  diagram.

Seeing  that  we  have  the  length  of altitudes and having information about the lengths of 


the  sides  which  the  altitudes are perpendicular to, we think about trying to split our big 
triangle  into  smaller  triangles  with  the  altitudes  that  we  have  already  draw.  We  can 
easily  do  so  by connecting the incenter to the vertices of the triangle. We will also let the 
sides of the triangle have lengths of a, b, and c,n such that a+b+c=p. 

39
 

From  this  diagram,  it  should  be  clear  that  the  lines  from  the  incenter  to  the  vertices  of 
the  triangle  split  our  original  triangle  up  into  3  smaller  ones.  Using  the  formula  Area=
,  we  get  the  area  of  our  smaller  triangles  to  be  ,  ,  and  .  Thus,  our  original 

triangle  has  an area of  + + = = . You will often seen this formula referred 


to by letting s, the semi perimeter, be equal to , and thus achieving . 

The  formula  for  the  area  of  a  triangle  given  above  is  just  one  of  the  many  helpful 
formulas.  Below,  we  will  provide  a  list  of  formulas  to  know  and  a  brief  idea of how to 
prove  each.  We  will  let  [ABC]  be  the  area  of  the  triangle,  r  be  its  inradius,  s  be  its 
semiperimeter,  R  be  its  circumradius,  and  a,  b,  and  c  be  the  lengths of its sides. We will 
also  let  the  triangle  have angles with measures of A, B, and C where angle A is opposite 
side BC, angle B is opposite side AC, and angle C is opposite side AB. 

40
Formulas: 

● [ABC]=   (draw  a  rectangle  around  the  triangle  and  look  at  the  two 
parts the altitude splits the triangle into separately) 

● [ABC]=rs (proved above) 

● [ABC]= (will be proved in trigonometry lesson) 

○ Note  that  this  formula implies the if two triangles have a congruent 


angle,  then  the  ratio  of  the areas of the triangles is equal to the ratio 
of  the  product  of the length of the sides of the triangle that form the 
congruent  angle.  This  can  be  easily  proved  with  the  definition  of  a 
sine,  or  the  knowledge  that  the  sine  of  an  angle  is  always the same 
(the  definition  of  basic  trigonometric  functions  will  be  given  in  the 
trigonometry lesson). 

● [ABC]=   (proved  easily  using  the  Extended  Law  of  Sines,  motivation 
being that we see the circumradius in the formula) 

● [ABC]=   (called  Heron's  Formula,  proved  by 


dropping an altitude, using Pythagorean Theorem, and lots of algebra) 

Sample  Problem  2.  Find  the  length  of  the  altitude  to  the  longest 
side of a triangle whose side lengths are are 4, 13, and 15. 

Because  we  know  that  Area= ,  we  could  solve  for  the  length  of  the  altitude  to  the 
longest  side  if  we  knew  the  area  of  the  triangle  (b=15,  h  is  the  length  of the altitude we 
want  to  solve for). However, using Heron's Formula, we can find the area of the triangle 

to be . Thus, we get , giving . 

41
This  problem  introduced  the  important  concept  of  finding  the  area  of  a  triangle  as  an 
intermediate  step,  then  using  the  area  and  other  knowledge  to  find  another value. This 
is  where  knowing  all  of  the  formulas  can  be  helpful  because  you  will  often need to use 
two  formulas  --  one  to  find  the  area  and  one  to  use  the  area  you found -- and you need 
to know the right formulas for the information you are given. 

42
Exercises 
3.1.1)  Find  the ratio of the inradius of a triangle to the inradius of the triangle formed by 
connecting that triangle's midpoints.  

43
3.2 Circles  
We  will  start  this  lesson  with  a  list  of  important  theorems  and  what  they  say.  Unlike in 
most  lessons,  we  will not fully prove all of them here, as there are a great many of them, 
so  we  encourage  you  to  try  to  prove  the  remaining  theorems  yourself  and  to  look  up 
proofs  online  if  you can't. In addition, because there are so many theorems and we want 
this  page  to  be  organized,  the  lesson  will  be  presented  slightly differently. Even though 
all  of  these  theorems  aren't  in  sample  problem  format,  we  encourage  you  to  attempt  to 
prove  them  yourself  before  reading  the solution, and we encourage you to try to look at 
the  methods  used  to  prove  these  theorems  as  general  strategies  that  can  be  used  in 
many  problems  involving  circles  (ex.,  extending  radii  out  to  tangents  is  always  a  good 
idea).  Finally,  the  names  of  these  theorems  are  not  that  important  to  memorize, so long 
as you are familiar with what the theorem does. 

44
Inscribed Angle Theorem 

This  theorem  tells  us  that  an  angle inscribed in a circle (its vertex and sides intersect the 


circle)  has  an  angle  measure  half  that  of  the  arc  it  intercepts  (the  arc  formed  by  the 
intersection  points  of  the  circle  and  the  side  of  the  angle). As seen in the diagram to the 
left,  we  would  have  that    (the  measure  of  the central angle, the one 
formed  by  connecting  the  arc's  endpoints  to  the  center  of  the  circle.  is  equal  to  the 
measure  of  the  arc).Note  that  this  theorem  also  implies  any  two  angles  intersecting  the 
same arc are congruent. 

We  can  prove  this  theorem  by  drawing  the  diameter  of  the  circle  through  the  vertex  of 
the  inscribed  angle  and  doing  casework  on  if  the  endpoints  of  the  arc  are  on  opposite 
sides  of  the  diameter,  one  is  on  the  diameter  and  the  other  isn't,  or  if  both  are  on  the 
same  side  of  the  diameter.  The  first  to  proofs are fairly trivial and for the third proof we 

45
can  use  our  second  proof  to  make  it  fairly  easy  as  well.  We  encourage  you  to  work out 
the details yourself.  

A  bunch  of  theorems  about  secants,  chords,  tangents,  and  angle 


measures with insignificant names 

Some  theorem  states  that  the  measure  of  the  angle  a  chord  forms  by  intersecting  a 
tangent  at  the  point  of  tangency  is  half  the  angle  of  the  arc  in  intercepts.In the diagram 
to the left, this would mean that .  

To  prove  this  theorem,  we  will  start  by  noting  that  CB  is  clearly  the  diameter  of  the 
circle  because  it  is  perpendicular  to  the  tangent.  Thus,  the  measure  of arc BE is equal to 
.  Note  that,  because  triangle  BAE  is  isosceles,  . 

46
Thus,  .  However,  this  means  that  . 
Thus, because BE was an arbitrary chord, we have proved the desired theorem. 

Another  theorem  states  that  the  measure  of  the  angle  that  two secants form where they 
intersect is equal to half the difference in angle measures of the two arcs in the circle that 
the  secants  intersect.  For  example,  in  the  diagram  to  the  left, 

We  will  prove  this  theorem  with  the  configuration  where  the  center  of  the  circle  is  in 
between  the  secants  and  other  configurations  can  be  handled  similarly  (although  we 
encourage  you  to  prove  them  on  your  own).  First of all, it is apparent that quadrilateral 
FAED  has  two  angles  that  we  want  to  know  about,  so  we  want  to  find  all of the angles 
in  the  quadrilateral  in  terms  of  angles  we want to end up using. Looking at angles AFD 
and  DEA  we  see  that  we  can  easily  relate  them  to  angles  GAE  and  HAF  using  the  fact 
that  triangles  GAE  and  HAF  are  isosceles.  Next,  we  note  that  even  though  GAE  and 
HAF  aren't  angles  we  want  to  end  up  in  our  answer,  the  fact  that 

47
  could  help  us  get  our  answer 
in  terms  of  the  angles  we  want.  Note  that 
. Similarly,  . 
Thus, 

The  same  theorem  holds  true  for  the  a  tangent  and  a  secant  that  intersect  outside  of  a 
circle  as  well  as  a  two  tangents  that  intersect  outside  of  a  circle.  The  theorems  say 
essentially the exact same thing and can be proved in a very similar way. 

A  theorem  states  that  when  two  chords  intersect  each  other  inside  of  a  circle,  the 
measure  of  the  angle  of  their  intersection  is  equal  to  the  average  of  the  measure  of  the 
arc  that  the  angles  intersects.  In  the  diagram  to  the  left,  this  would  mean  that 

48
You will prove this theorem in the exercises. 

Power of a Point 
This  theorem  has  a  different  definition  than we will cover here, but as far as your use of 
it will go, this definition will serve to be all you need. The theorem has several parts. 

In  this case, the theorem states that if we draw any chord through a point in a circle, The 
product  of  the  distance  from  that  point  to  the  two  intersections  of  that  chord  with  the 
circle  will  stay  constant.  In  the  diagram,  the  theorem  tells  us  that 
.  

To prove this theorem, we can just draw two arbitrary chords through an arbitrary point 
and  prove  that  these  for  these  to  chord  the  theorem  holds.  This  will  prove  the  whole 
theorem  because  this  means  that  for  any  point,  if  we  have  one  chord,  the  desired 

49
product  will  be  equal  to  the  product  for  that  chord  in  any other chord that we draw. To 
prove  that  this  holds,  we  will  use  the  picture  to  the left as our diagram. We can see that 
triangles  EBF  and  CDF  are  similar  using  the fact that angle BEF and DCF both intercept 
the  same  arc-and  are  thus  congruent  by  our  previous  our  previous  theorem  and angles 
EBF  and  CDF  are  similarly  congruent.  Thus,  we  get 

In  this  case,  this  theorem  would  state  that  For any two secant to a circle that intersect at 


a  point  outside  of  the  circle,  the  product  the  the  distances  from  the  point  outside  the 
circle  to  the two intersection points of the secant going through hath point and the circle 
is equal. In the diagram to the left, this would mean that . 

You will prove this in the exercises. 

50
 

Next,  in  this  case,  the  theorem  states  that the product of the distances from the intersect 


of  a  tangent  and a secant to a circle to the two intersections of the tangent with the circle 
is  equal  to  the  distance  from  the  same  intersection  point  to  the  point  of  tangency 
squared. In the diagram to the left, this theorem would state that . 

51
To  prove  this  case,  we  will  introduce  a  concept  called  the  power  of  a  point,  the  very 
concepts this theorem is named after. The power of point P with respect to circle O is the 
distance  from  P  to  O  squared  minus  the  length  of  the  radius  of  circle  O  squared.  Note 
that  this  quantity  can  be negative when P is inside circle O. For this proof, we will show 
that  both  of  our  desired  expressions  are  equal  to  the  power  of  point  D  with  respect  to 
circle  A.  First  of  all,  by  the  Pythagorean  Theorem,  we  have 
.  Thus,    is  equal  to  the  power  of 
point  D  with  respect  to  circle  A.  Next,  note  that 
.  Thus,    is  equal  to  the  power 
of  point  D  with  respect  to  circle  A  and  we  are  done.  Note  that  this  implies  that  if  two 
tangents  intersect  at  a  point  outside  of  a  circle,  then  the  distance  from  that  point  to  the 
points of tangency is the same for each tangent. 

52
Exercises 
3.2.1) Prove Theorem 3 (its diagram is labeled Thm. 3 in red). 

3.2.2) Prove the second case of the Power of a Point theorem. 

 
 
 
 
 
 
 

53
3.3 Analytical Geometry  
Before  we  start  the lesson, we will go over when to use analytic geometry and when not 
to.  Analytic  geometry  rarely  produces  pretty  solutions  and  is prone to arithmetic errors 
and  the  like  in  the  algebra. Thus, we encourage you to not jump to analytic geometry to 
solve  ever  geometry  problem.  Now,  this  isn't  to  say  that  you  should  never  use  analytic 
geometry.  It  is  a  very  useful  tool  to  have  and  can  solve  many  problems  easily that may 
have  otherwise  been  very  hard.  Analytical  geometry  is  especially  effective  when  there 
are  lots  of  lengths  given,  several  right  angles,  and  the  intersection  of  lines.  However, 
even  on  most  problems  that  do  seem  to  fit  these conditions, we encourage to try to find 
a non-analytic (synthetic) solution first, as these solutions are often much quicker. 

We  will  start  by  introducing  a  theorem  which  we  will  not  prove.  There  is a proof using 
induction  and  the  cross  product  which  is  too  advanced  for  the  curriculum.  However, 
this  theorem is quite necessary for many problems that use "coordinate bashing" as their 
solution.  

54
The Shoelace Formula 

This  formula  provides  a  way  to  find  the  area  of  any  polygon  given  its  vertices  in  the 
order they are connected. The best way to explain it is to show a visual. 

55
To  make  this  clearer,  we  will  go  over  an  example.  Say  that  we  had  a  triangle  with 
vertices  (1,-2),  (7,8),  and (1,1). Note that if this weren't a triangle we would have to draw 
a  quick  sketch  to  get  the  order  of  the  points,  but  for  a  triangle  any  ordering  will  be 
correct  because  each  point  is  connected  to  the  other  two.  Thus,  we  write  out  the points 
in order. 

(1,-2) 

(7,8) 

(1,1) 

(1,-2) 

From  here  we  get  the  area  to  be 


.

Sample Problem 1. Find the area of triangle ABF given that E is 

the midpoint of CB. 

56
In  order  to  show  how  to  coordinate  bash  a  problem,  the  best  way  is  to  show  an 
example.  This  problem's  solution  will  contain  many  important  techniques  that  you 
should remember.  

First  of  all,  we  will  start  by  setting  our  coordinate  axis.  We  let  A=(0,0),  B=(1,0),  and 
D=(1,0).  Thus,  E=(1,0.5).  Next,  using  the  shoelace  formula,  we  know  we  can  find  the 
desired  area  if  only  we  had  the  coordinates of F. However, we know that we can do this 
by  finding  the  intersection  of  two  lines.  We  can  note  that  line  AE  has  the  equation 
.  We  can  also  note  that  line  BD  has  the  equation  .  Thus,  setting  the 
two  lines  equal  and  solving  we  get  that  their  intersection  is  .  Now,  we  could  use 
the  shoelace  formula  to  find  the  triangle's  area,  but  in general we should look if there is 
a  quicker  way  first.  In  this  case,  now  that  we  have the y coordinate of F, we know  that 
AB  has  length  1  and  the  height  draw  to  AB  has length 1/3. Thus, triangle ABF has area 

.  An  important  note  is  that  while  coordinate  bashing,  we  can  use  other 
formulas or some synthetic (non-coordinate) techniques to reduce our calculations. 

As  you  saw  in  this  problem,  the  key  to  analytical  geometry  is to find the coordinates of 
points.  you  saw  several  useful  methods  of  how  to  do  this  in  the  problem,  but  we  will 
create a more extensive list below for you to use when solving problems. 

How to find the coordinates of a point 

● Find  the  equations  of  two  lines  (or  any  other  shape)  that  the  point  is  on 
and compute the intersection of the lines to find the point. 

● Find  a  point  that  the  point  you  are  given  is  directly  above  or  directly 
horizontal  to  for  which  you  know  the  distance  between  these  two  points. 
Further,  if  you  know  that  a  point  is  some  number  to  the  left  or  right  and 
some  number  up  or  down  from  another  point  whose  coordinates  you 
know, you can easily find that points coordinates. 

● If  a  point  is  the  midpoint  of  a  line  segment,  then  its  coordinates  are  the 
average of those of that segment's endpoints. 

57
There  are  many  other  ways  to  find  a  point's  coordinates,  but  the  three  listed  above  are 
the ones that come up the most often. 

Finally,  when  you  are  coordinate  bashing  you  can choose any point to be the origin and 


any  lines  as  the  x  and  y  axis.  Thus,  you  should  make  sure  to  choose  an  origin  an  axis 
that  make  calculations  easier  (more  specifically,  choose  a  point  that  would  make  other 
points easier to find using the methods to find points provided above). 

58
Exercises 

3.3.1) Find the centroid of a triangle with coordinates A= ,B= , and C= . 

3.3.2)  Find  the  Area  of  triangle  ABE  given  that  E  is  the  intersection  of the quarter-circle 
centered at B through A and line BD. 

 
 

59
3.4 Three-Dimensional Geometry 
Sample  Problem  1.  Find  the  volume  of  the  octahedron  formed  by 
connecting the centers of the sides of a cube with edge length 1. 

At  first  it  seems  we  have  no  idea  where  to  start.  We  don't  even  know  how  to  find  the 
volume  of  the  if  we  find  one  of  its  side  lengths  (we  need  two).  Thus, we want to try to 
think  about  what  information  we  do  have,  preferably  in  2  dimensions  to  make it easier 
to  understand.  The  base  (the  middle  square-shapes  part)  of  the  octahedron  seems 
important,  so  we  will  look  at  a  cross-section  of  the  plain  that  it  is on. When we do that, 
will  will  see  that  it  is  simply  a  square  with  a  smaller  square  inside  it  that  is  formed  by 
connecting  the  midpoints  of  the sides of the square (the smaller square is the base of the 
octahedron).  The  larger  square  clearly  has  side  lengths  one  (as  the  cross-section  was 
parallel  to  a  face  of  the  cube),  so  we  find  that  the  base  of  the  octahedron  has  an area of 
1/2.  Next,  we  can  split  the  octahedron  into  two  pyramids  with  the  same base and each 

60
having  a height of 1/2. Using the area we just found of the base, we get each pyramid to 
have a volume of , so the total volume desired is . 

As  you  can  see  in this problem, the key to almost any 3d geometry problem is to split of 


3d  shapes  into  smaller  3d  shapes  and  to  take  2d  cross-sections  of  important  parts  of 
your  figure  in  order  to  gain  needed  information,  ex.  the  area  of the base. In fact, almost 
any  3d  geometry  problem  can  be  split  into  several  simpler  2d  geometry  problems  in 
order to be solved. 

As  you  saw  in  the last problem, it was important to know the formula for the volume of 


a  tetrahedron.  There  are  several  other  3d  volume  formula  that  are  important  to 
remember listed below. These formulas will not be proved, but you can gain an intuitive 
understanding  of  why  many  of these work by thinking about inscribing them  in a cube 
and comparing the shape's volume to the volume of the empty part of the cube. 

● Cube: V=  

● Tetrahedron, cone, pyramid: V=  

● Sphere: V=  

Other  than  these  main  volume  formulas,  most  other  shapes  can  be  made  of  these 
shapes, so you should be able to derive the volume without a formula. 

61
Sample  Problem  2.  Find  the  volume  that  the  shape  below  covers 
when  rotated    around  AB.

When  we  imaging  the  shape  that  this  forms  it  looks  like  two  circles,  one  with  radius  3 
and  one  with radius 1, that are 1 unit apart and how their circumferences connected. We 
don't  immediately  know  how  to find the volume of the shape, but it should remind you 
of  a  cone.  In  addition,  we  know  that  when  you  rotate  a  right  triangle around similar to 
how  we  rotated  the  above  shape  we  get  a  right  cone.  However,  comparing  the  shape 
above  to  a  triangle gives us the idea that for some point E on line AB we could draw EA 
and  EC  to  form  right  triangles  EAC  and  EBD.  In  other  words,  the  shape  above  is  just 
triangle  EBD  minus  triangle  EAC  for  some  point  E  on  line  AB.  However,  this  would 
also  mean  the  the  3d  shape  created by rotating the above shape is just the shape created 
by  rotating  triangle  EBD  minus  the  shape  formed  by  rotating  triangle  EAC.  However, 
the  shape  formed  by  rotating  a  triangle  is  just  a  cone,  and  we  know  how  to  find  the 
volume  of  cones! Thus, we just need to find the the distance EA to get the height of both 
cones.  Notice  that  for  E,  C,  and  D  to  be  concentric  we would need triangle EAC similar 

to  triangle  EBD.  Thus, we get  . Thus, we get the bigger cone 


to  have  height  of  3  and  the  smaller  to have height of 1. Therefore, the desired volume is 
.  The  shape  that  we  found  the  volume  of  is  called  a 
frustum. 

62
The  technique  used  here  of  thinking  of  one shape as a larger shape minus a smaller one 
is  important  to  remember  and  can  be  used  to  express  many  complicated  3d  shapes  in 
terms of simpler ones we are more familiar with. 

63
Exercises  

3.4.1) What is the volume of a cube inscribed in a sphere with radius . 

3.4.2)  What  is  the  volume of the shape formed by connected the centers of each face of a 


cube with side length of 1? 

 
 

64
3.5  Similar  and  Congruent 
Triangles 
Sample  Problem  1.  Given  that  AB  =  AC  and  that  BD  =  DC,  prove 
that AD is perpendicular to BC. 

First  of  all,  note  that  proving that AD is perpendicular to BC is the same as proving that 


  because  those  two  angles  add  to  180  degrees.  Seeing  information 
regarding  two  sides  of  triangles  ABD  and  ACD,  we  try  to find information on the third 
side  of  the  triangles  because  both  triangles  contain  one  of  the  angles  we  want  to  prove 
equal.  However,  we  can  easily  see  that  the  AD  is  the  remaining  side  of  both  triangles, 
and  it  is  obvious  that  AD  =  AD.  Thus,  we  note  that  all  three  sides of triangles ACD are 
congruent  to  the  corresponding  sides  of  triangle  ABD.  From  here,  we  can  see  that  this 
implies  that  triangles  ACD  and  ABD  are  congruent.  In  other  words,  the  triangles  are 

65
essentially  the  same  and  can  be  one  can  me  moved  onto  the  other  with  only 
translations,  rotations,  and  reflections  and  all  corresponding sides and angles are equal. 
Note  that  when  stating  a  congruence,  the  order  of  the  points  in  each  shape  matter 
because  they  are  used  to  show  how  the  two  shapes  map  to  each  other  (the  first  points 
named  by  the  first  letter  in  the  name  of each shape map to each other, and so on). Thus, 
we see that as desired, and we are done. 

The  claim  that  these  two  triangles  are  congruent  may  not seem obvious at first. Let's go 
into  more  detail  to  see  how  we could prove that. Here, we will prove it by showing that 
given  three  side  lengths,  any  triangle  constructed  with  those  side  lengths  must  be 
congruent.  To  do this we will start with translate and rotate any triangle satisfying these 
conditions  such  that  the  side  with the longest length is mapped onto AB. From here, we 
will  look  at  all  the  possible  triangles  that  could  have  been  created.  If  a  triangle  does 
have  the  above  three  lengths  and  longest  sides  at  AB,  then  we  have two possibilities in 
deciding  the  length  of the side that comes out of A. From there, we draw a circle from A 
with  a  radius  equal  to  the  length  of  the  side  coming  out of A and do the same for point 
B.  We  thus  see  that  the  third  point  of  our  triangle  is  the  point  on  the  intersection 
between  these  two  circles,  leaving  two  possibilities  for  where  the  point  is.  However, 
because  we  could  switch  the  lengths  of  the  sides  coming  out  of  A  and  B,  we  see  that 
there are actually 4 possible locations for the third point of our triangle. 

66
 

Finally,  as  seen  in  the  diagram  above,  once  we  have  one  of  the  triangles  corresponding 
to  a  possible  location  of  the  third  point,  we  can  reflect  it  over  AB,  the  perpendicular 
bisector  of  AB,  and  both lines, in order to produce three more, for a total of 4, congruent 
triangles  corresponding  to a possible location of the third point. However, because there 
are  only  4  possible  locations  for  the  third  point,  as  described  above,  we  have  just 
covered  each  of these possible locations and showed that each creates a triangle that can 
be  mapped  on  to  each  other  triangle  with  only  reflections.  Thus,  we  have  proved  that 
any  triangle  with  three  side  lengths  is  congruent,  and  we  are  done.  If  you  don't 
understand this proof, go over it again until you do, because it is very important. 

Now  that  you've  seen  that  we  can  prove  two  triangles  congruent  without  explicitly 
proving  all  of  their  sides  and  all  of  their  angles congruent, you may wonder if there are 
other  sets  of  givens  that  can  prove  two  shapes  congruent.  Indeed,  two  triangles  can  be 
proved  congruent  in  several  ways,  which  will  be  listed  below.  For each such method of 
proving  two  shapes  congruent,  try  to  convince  yourself  that  any  two  triangles  sharing 
these  characteristics  must  indeed  be  congruent.  Finally,  try  to  remember  the  list  given 

67
below.  Congruent  triangles  are  the key to many geometry problems, so you will need to 
know what to look for to prove two triangles congruent. 

Possible ways to prove triangles congruent 

● Two  triangles  are  congruent  if all 3 pairs of sides are congruent (called SSS 


congruence). 

● Two  triangles  are  congruent  if  they  have  two pairs of congruent sides and 


the  angle  between  those  two  sides  is  also  congruent  in  both  triangles 
(called SAS congruence). 

● Two  triangles  are  congruent  if  they  have  a  right  angle  and  their 
hypotenuses  as  well  as  one  pair  of  corresponding  legs  are  congruent 
(called HL congruence). 

● Two  triangles  are  congruent  if  two  angles  and  the  side  between  them  are 
congruent (called ASA congruence). 

These  methods  of showing two triangles congruent can be used to show that two angles 


or  lengths  are  equivalent  by  proving that those two lengths or angles are corresponding 
parts of congruent figures. 

68
Sample  Problem  2.  Using  the  given  lengths  in  the diagram below, 
find AD. 

For this problem it isn't immediately obvious what to do, so we will start by seeing if we 
can  find  any  additional  lengths  using  the  given  information.  Because  ABA'  is  a  right 
triangles,  we  can  use  the  Pythagorean  theorem  to  see  that  AA'  =  5.  Seeing that we now 
have  a  lot  of  information  about  lengths  for  triangles  ABA',  we  will  try  to  see  if  we  can 
relate  this  triangle  to  some  triangle  that  contains AD in order to try to find the length of 
AD.  Notice  that  because  angle  A  is  congruent  to  itself,  ,  and 
the  angles  of  a  triangle  add  to  180  degrees, the angles of triangle ACD are all congruent 
to  corresponding  angles  of triangle ABA'. Thus, these two triangles seem highly related, 
but  they  definitely  aren't  congruent.  These  triangles  are  similar  triangles,  which  means 
that  you  can  move  one  on  to  the  other  using  translations,  rotations,  reflections,  and 
scaling/dilation.  We  can  see  that,  by  changing  the  length  of  each  side  of  triangle  ACD 
by  moving  points  C  and  D along sides AB and AA' respectively, we can essentially map 
triangle  ACD  onto  triangle  ABA'  because  CD  is  parallel  to  BA'.  This  transformation  is 
called  a  dilation.  Because  CD  maps  to  BA',  we  can  see  that  this  dilation  mapping  ACD 
to  ABA'  doubles  the  lengths  of  each  side  of  triangle  ACD.  You  can see that AC and AD 
must  be  scaled  the  same  amount  because  CD  is  parallel  to  A'B,  so,  using  the 
Pythagorean  theorem,  we  see  that  doubling  A'B  requires  doubling  the  other lengths. In 

69
order  to  generalize  this  for  other  problems  that  aren't  just  right  triangles  being  dilated, 
think of splitting triangles into two right triangles, or expressing an acute triangle as one 
right  triangle  minus  another,  as  seen  in  the  diagram  below.  Thus,  we  can  conclude  AD 
is half of AA' and has a length of 2.5. Make sure you understand this solution, because it 
is very important. 

There  are  several  ways  to  prove  two  triangles  similar.  For  similar  triangles  ABC  and 
DEF,  it  can  be  shown  that  AB/DE=BC/EF=AC/DF  and  that  angles  A  and  D,  B  and  E, 
and  C  and  F  are  all  congruent.  Knowing  that  two  triangles  are  similar  can  be  used  to 
find  lengths  or  angles,  as  in  the  problem  above.  Thus,  it  is  important  to  know  the 
criteria  for  two  triangles  to  be  similar.  A  list  of  ways  to  show  two  triangles  similar  is 
given  below,  and  for  each  way  you  should  try  to  convince  yourself  that  the  given 
conditions do indeed make two triangles similar. 

Ways to show two triangles similar 

● Two  triangles  are  similar  if  all  three  of  their  corresponding pairs of angles 
are  congruent  (two  pairs  of  corresponding  angles  being  congruent  is 
enough  because  all  angles  of  a  triangle  add  to  180  degrees).  This  is  called 
AA similarity. 

● If  three  pairs  of  corresponding  sides  all  have  the  same  ratio  between  the 
two triangles, then the triangles are similar (called SSS similarity). 

● If  two  pairs  of  corresponding  sides  have  the  same  ratio  between  triangles 
and  the  angle  between  them  is  the  same  for  each  triangle,  then  the  two 
triangles are similar (called SAS similarity). 

70
Note  that  the  list  of  ways  to  show  two  triangles  similar  is  very  similar  to  the  ways  to 
show two triangles congruent, suggesting a similarity in proofs. 

71
Exercises 
3.5.1)  Prove  that  the  ratio  between  each  pair  of  corresponding  sides  is the same for two 
triangles who have all of their corresponding angles congruent. 

3.5.2) Find EF in terms of FB and FC. 

 
 

72
3.6 Trigonometry 
We will start this lesson off with some definitions of basic trigonometric functions. 

The Sine 

The  sine  of  an  angle    is  written  as    and  is  the  distance  from  the  intersection  of 
the  radius  of  the  unit  circle  that  has  been  rotated    counter  clockwise  about  the  origin 
from  the  x-axis  and  the  diameter  of  the  unit  circle  to  the  x-axis  (as  can  be  seen  in  the 
diagram  above).  Note  that  when  this altitude to the x-axis is below the x-axis the sine of 
the  angle  is  negative.  When    is  between    and    or    and    (if  you  don't 
know  what  the  radian  is  you  should  look  it up because it is used very commonly), then 
  is  positive.  In  addition, when   is between   and  ,   can be viewed in 
the  context  of  a  right  triangle  as  the  ratio  of  the  length  side  opposite  the  angle  to  the 
length  of  the  hypotenuse  (think  about  how  the  radius  of  the  unit  circle  is  the 
hypotenuse  of  the  triangle  in  the  first  definition  and  how  from  there  we  can  scale  it up 
for larger hypotenuses without changing the value of the sine). 

The Cosine 

The  cosine  of  an  angle   is written as   and is the distance from the intersection of 


the  radius  of  the  unit  circle  that  has  been  rotated    counter  clockwise  about  the  origin 
from  the  x-axis  and  the  diameter  of  the  unit  circle  to  the  x-axis  (as  can  be  seen  in  the 
diagram  above).  Similar  to the sine, the cosine is negative when the point is to the left of 
the  y  axis  (i.e.  for  ).  In  addition,  for  angles  angles between   and  , 
the  cosine  can  be  seen  in  the  context  of  a  right  triangle  as  the  ratio  of  the lengths of the 
side  adjacent  to the angle over the hypotenuse of the triangle (again, think about scaling 
up the unit circle).  

The Tangent  

The  tangent  of  an  angle    is  written  as    and  is  the  length  of  the  line  segment 
perpendicular  to  the  radius  of  the  unit  circle  that  has  been  rotated    counter-clockwise 
about  the  origin  from the x-axis with endpoints on the x-axis  and the intersection of the 

73
aforementioned  radius  and  the  unit  circle's  diameter.  The  tangent  is  negative  when 
exactly  one  of  the  sine cosine is negative. The tangent can also be seen as  . Thinking 
about  the  right  triangle  definitions  of  sine  and  cosine,  we  can  get  that  for  angles 
between    and  ,  the  tangent  in  a  right  triangle  is  equal  to  the  ratio  of  the  side 
opposite the angle to the side adjacent to the angle. 

There  are  a  few  other  trigonometric  functions  and  other  ways to define the sine, cosine, 


and tangent, but these are far beyond the scope of the AMC 10. 

There  are  a  few  important  trigonometric  identities  and  values  that  you  need  to  know. 
This  list  will  not  include  the sum and product identities because they are not needed on 
the  AMC  10,  but  you  should  look  them up if you are interested: they provide proofs for 
some  trigonometric  ratios  that  you  should  memorize.  Also  note  that  the  square  of  a 
trigonometric  value is placed before the parentheses. For example, we write   and 
not . 

●  

●  

●  

●  

●  

●  

● (think of the unit circle) 

●   

● ,  this  is  apparent  if  you  think  of  the  unit 


circle 

74
●   

●  

●  

●  

●  

●  

Sample Problem 1. Prove that . 

For  this  problem,  when  we  seen  that  one  angle  is    minus  another,  we  think  of  the 
angles  of  a  right  triangle,  especially  because  right  triangles  have  so  much  to  do  with 
trigonometry.  By  looking  at  a  right  triangle  and  seeing  how  two  two  non-right  angles 
add  to  ,  we  can  easily  confirm  this  identity  to  be  true  for   (

).  We  will  proceed  by  casework  on  the  quadrant  that angle   is 
in.  If  it  is  in  the  second  quadrant,  then  we  can  say    for  . 
Thus, 


For  the  third  and  fourth  quadrants,  note  that 


Thus, by building on each previous result, we are done. 

75
 

Now,  you  may  be  thinking  that  it  this  is  nice  and  all  and  will  help  find  lengths in right 
triangles,  but  how  often  do  we really use right triangles? Well, we do use right triangles 
pretty  often,  but  we  can  extend  this  beyond  right  triangles.  We can use trigonometry to 
help us in any triangle. 

Sample  Problem  2.  In  triangle  ABC  let  BC=a,  AC=b,  AB=C, 
,  ,  ,  and    be  the 
radius  of  the  circumscribed  circle.  Prove  that 

Knowing  that  we  are  dealing  with  right  triangles  and  that  we  are  dealing  with  a 
circumradius,  we  will  want  to  draw  the  circumcircle  of  our  triangle  (remember  that 
because  any  angle is half the arc it intersects, any angle that intersects the endpoints of a 
diameter  will  be  right).  There  doesn't  appear  to  be  any  reason  to  to  draw  the  diameter 
through  any  vertex  of  the  triangle  as  opposed  to  another  one,  so  we  try  drawing  the 
diameter  through  the  middle  vertex.  We  connect  the  point  the  point  on  the  opposite 

76
endpoint  of  the  diameter  we  just  drew  to  one  of  the  vertices  because  we  then  get  an 
angle  intersecting  the  same  arc  as  one  angle  of  our  triangle  does.  From  here we can see 
that  .  We  can  also  see  that    because  the  angle 
intersects  the  diameter  of  our  circle.  Thus,  we  can  get 

For  any  acute  triangle,  we  would  simply  repeat  this  process  for  each  angle  to  get  the 
desired  result.  However,  because  we  have  an  obtuse  triangle,  we  need to do something 
different  for  the  obtuse  angle,  .  What  we  did  worked  well,  so  we  want  to  do 
something  similar.  We  want to have an angle that is at least intersecting the arc opposite 
the  arc    intersects  (using  the  fact  that  an  inscribed  angle  is  half  the  measure  of  the 
arc  intersects  we  get  that  the  angles  add  to  ,  and  we  know  that 
).  Similar  to  how  we  did  last  time,  we  connect  the  point  on  the 
opposite  endpoint  of  the  endpoint  we  just  drew  to  point  .  We  again  have  that  angle 

BCG  is  a  right angle. From here we get that  . 


This  theorem  is  trivial  to  prove  for  right  triangles.  Thus,  we  are  done. This relationship 
we just derived is called the Extended Law of Sines. 

77
 

78
Another  relationship  we  can  get  using  trigonometry  is  that,  using  the  same  labeling 
system  as  the  previous  problem,  .  This  is  called  the  Law  of 
Cosines.  We  will  not  prove  this  theorem,  but  we  encourage  you  to  try  to  prove  this  on 
your  own  (Hint:  What  theorem  does  this  look  like?  What  do  we  always  often  use  in 
trigonometry?) or to look up a proof.  

79
Exercises 

3.6.1) Prove that . 

3.6.2) Prove the values given for the sine of and . 

80
 
 
 
 
 
 
 
 
 

81
4 Number Theory 
4.1 Euclidean Algorithm 83 
4.2 Modular Arithmetic Part I 87 
4.3 Modular Arithmetic Part II 92 
 

 
 
 

82
4.1 Euclidean Algorithm 
Sample  Problem  1. What is the greatest common divisor of 388899 
and 388877 (find it without a calculator)? 

We  want  just  go  ahead  and  factor  both  numbers  and  compare  factorization,  but  clearly 
that  won't  work.  Who  would  want  to  factor  those  huge  numbers?  There  must  be  a 
better  way.  Well,  these  two  numbers  do  appear  to  be  fairly  close together, so maybe we 
would  do  something  with  their  difference,  or  some  other  smaller  quantity.  Well,  if 
,  then  we  get  that    and    for 
integers , and .  

But  this  gives  us  exactly  what  we  wanted!  This  lets  us  realize  the 
  because  .  Thus, 
we  have 
.  Now, 
we  simply  can  check  to  see  if  388877  is  divisible  by  2  or  11.  It  is  clearly  not  even,  and 
using  the  trick  to  see  if  a  number  is  a  multiple  of  11  we  find  that  3+8+7-8-8-7=-5,  so 
388877  is  not  a  multiple  of  11.  Thus,  these  two  numbers are relatively prime or, in other 
words,  have  a  greatest  common  divisor  of  1.

This  method  that  we  just  saw  seems  pretty  useful  in  helping  us  find  the  greatest 
common  divisor  of  two  large numbers. But, what if  our numbers don't have as small or 
easy  to  test  out  of  a  difference?  Let's  try,  for  example,  finding  the  greatest  common 
divisor  of  71  and  193.  Now,  these  numbers  are  smaller  and  we  probably  could  factor 
them,  but  the  point  here  is  to  explore  our  previous  method  and  to  see  if  we  can  take  it 
all  the  way  to  the  answer  that  we  want.  We  will  start  the  same  as  we  did  last  time. 
.  However,  the  first  number  is  still bigger than the second 
number.  This  brings  us  to  an  important  concept:  using  the  exact  same  reasoning  as  for 
one  subtraction,  we  don't  just  have  to  subtract  once;  we  can  subtract  one  number  from 
the  other  until  our  first  number  is  smaller  than  the  second!  In  other  words,  we  can 
replace  a  number  with  its  remainder  when  divided  by  the  other  number  (note that this 

83
just  repeated  subtraction,  and  sometimes  repeated  subtraction  is  easier  than  actually 
trying to divide the number). Therefore, we have . 

We  could  again  easily  factor  here, but we still want to try to discover a whole process to 


try  to  get  us  to  the  end  (we  are  experimenting  with  smaller  numbers  because  they  are 
easier  to  think  about,  but  we  want  to  make  a  process  that  will work for larger numbers 
as  well).  Well,  now  we  are  just  back  where  we  started,  but  with  slightly  smaller 
numbers.  However,  this  is  exactly what we wanted! We can just repeat what we just did 
over  and  over  again  to  get  to  smaller  and  smaller  numbers.  Let's  use  this  to  finish  out 
our  problem: 

Now,  we  could  stopped  this  process  much  earlier  and  easily  seen  our  answer,  but  we 
continued it on until we had a 0 to show something: we did end up being able to get a 0. 
In  fact,  if  ,  then  we  can  get  use  our  method  to  get    to   
(remember  that  0  is  a  multiple  of  every  number  because  any  number  times  0  equals 0). 
Now,  you  might  be  thinking,  that's  nice  and  all, but why do I need to know that we can 
always  reach  a  zero  to  find  the  greatest  common  divisor  of  two  numbers?  Couldn't  I 
have  stopped  much  earlier,  have  done  less work, and have gotten answer just as easily? 
The  answers  to  these  questions  are  that  you  don't need to know this to find the greatest 
common  divisor,  yes  you  could  have  stopped  earlier,  yes  you  could  have  could  have 
done  less  work,  and  yes,  if  you  wanted  to  find  the  greatest  common  divisor,  you 
probably  should  have  stopped  earlier.  But,  isn't  finding  the  greatest  common  divisor 
exactly  what  this  algorithm  was  designed  to  do?  The  answer,  again,  is  yes, that is what 
we  designed  it  to  do, but just because we designed an algorithm to do one thing doesn't 
mean  it  can't  do  another  (now  this  isn't  to  say  that  you  shouldn't  use  this  algorithm  to 
find  the greatest common divisor of large numbers; you should, as this algorithm makes 
it  much easier). We will see another way to use this algorithm we just discovered, which 
is  called  the  Euclidean  Algorithm  as  you  may  have  guessed.  in  the  Diophantine 
equations lesson. 

Just  to  make  sure  you  understand  the  Euclidean  Algorithm  fully,  we  will  go  through 
another full example of it. 

84
 

Sample Problem 2. Find . 

Using  the  Euclidean  Algorithm,  we  get 


.  Now, 
we  could  stop  here  and  just  see  that  their  greatest  common  divisor  is  2,  and  for  this 
problem,  that  would  be  perfectly  correct.  However,  just  to  make  sure  you  understand, 
we will finish off this example.  . Notice that, 
as expected, we do end up getting to a 0 and a 2 in the end. 

85
Exercises 

4.1.1)  Prove  that  if  ,  then  we must be able to use the Euclidean Algorithm 


to get our down to . 

4.2.1)  Using  the  Euclidean  Algorithm,  express  1  in  the  form    for  integral 
(not  necessarily  positive)    and    (do  not  guess  and  check;  use  the  Euclidean 
Algorithm in a way that you are guaranteed to find a solution). 

 
 
 
 
 
 
 

86
4.2 Modular Arithmetic Part I  
We  will  start  with  definitions.  First  of  all,  and  most  basically,  we  will  go  over  the 
definition  of  congruence  in  a  modulo.  We define   to mean that   is 
divisible  by  .  We  say  this  as    is  congruent  or  equivalent  to   in  . In other 
words,  it  means  that    and    have  the  same  remainder  when  divided  by  .  For 
example,    because  10,  4,  and  1  all  have  a  remainder  of  1  when 
divided  by  3.  We  will  talk  about  multiplication,  division,  addition,  subtraction,  and 
exponentiation  in  the  lesson.  Note  that  we  let    as  well.  We  call 
the modulus. 

Sample  Problem  1.  If  some  number  of  people  are  separated  into 
rows  of  10,  there  are  3  left  over.  How  many  people  are  left  over  if 
3  more  people  are  arranged  into  the  rows  of  10?  What  does  this 
have to do with the definition we just discussed? 

In  this  problem,  it  is  clear  that  we  aren't  doing  anything  to  the  rows  of  10,  but  just 
adding  3  more  people  to  the  people  left  over.  Thus,  we  have  6  people  left  over.  This 
seems  to  be  common  sense,  and  it  is.  Let's  see  how  this  applies  to  modular  arithmetic. 
First,  we will start by rewriting the problem in terms of the terminology we just saw. We 
will  let  the  number  of  people be  . Thus, we see that  . Rewriting what 
we  found  out  about  adding  in  3  people,  we  see  that  . 
Now, this is no proof that addition works as normal in modular arithmetic, but it should 
be  fairly  clear  from  thinking  about  how  we  solved  this  problem  that  you  can  add 
anything to both sides of an equivalence. 

87
Sample  Problem 2. Does multiplication work like it normally does 
in  modular  arithmetic?  In  other  words,  can  you  multiply  both 
sides  of  a  congruence  by  the  same  thing  in  modular  arithmetic? 
How about exponentiation? 

We  can  fairly  easily  see  that  multiplication  does  indeed  work  as  normal  in  modular 
arithmetic.  Too  prove  this  we  just  need  to  remember  that multiplication is just repeated 
addition.  Because  we  just  saw  that  addition  works  in  the  previous  problem.  Using 
similar  reasoning,  we  can  see  that  exponentiation  works  because  it  is  just  repeated 
multiplication.  

Sample Problem 3. Does division work in modular arithmetic? 

You  might  think  that  the  answer  is  yes,  but  having  fractions  sometimes  result  from 
divisions  should  give  you a pause. After all, the very definition of modular arithmetic is 
based  on  working  with  solely  integers.  The  answer  to  the  question  ends  up  being,  yes, 
kind  of,  and  weirdly.  Let’s  look  at  a  few  examples  to  see  how  we  think  division  would 
work.  Let's  say  that  we  have  .  If  we  divide  both  sides  of  the 
congruence by 7, we get  . While   may appear to work 
at first, and it is a solution, there are other solutions that work, such as 
.  Thus,  unlike  the  other  operations  we  saw,    does  not  imply 
(at  least  not  always)  even  though    does  imply 
.  Also,  be  careful  in  that  dividing  doesn't  always  produce  a  true 
statement.  For  example,  ,  but  .  In  addition,  division 
seems  weird  without  being  able  to  do  it  to  every  number (we can't get fractions), so we 
can  conclude  that  division,  in  the  traditional  sense,  does  not  work  in  modular 
arithmetic.  

We  did  see  that  one  of  the  divisions  produced  a  true statement, and if we want to solve 
equations  similar  ,  we  will  need  to  be  able  to  "divide."  Let’s  try  to 
think  about  what  division  does  in  order  to  try  to  come  up  with  something  that  acts 
similarly.  First  of  all,  let's  look  at    means  in  algebra. using our earlier 
definitions.  Thinking  about  how  it  means  that  both  sides  of  the  congruence  have  the 
same  remainder,  we  can  rewrite  our  equation  as  7a=7+49y  (y  is  integral,  but  not 

88
necessarily  positive).  Thus,  when  we  divide  this  equation  by  7,  we  get a=1+7y. In other 
words,  by  converting  back  to  modular  notation,  we  get  .  Generalizing 
what we just saw, we get one form of "division" in modular arithmetic:   
implies   if  is  a  divisor  of  ,  ,    (otherwise  we  get  fractions  and 
fractions don't work with remainders and modular arithmetic). 

Now  that  we  have  defined  "division"  in  some  cases,  what  will  we  do  if  all  of  the 
numbers  don't  share  a  common  divisor?  For  example,  how  do  we  find  all  of  the 
solutions to or . 

We  will  look  at  cases  similar  to  the  former  example  first;  we  will  look  at  cases  of 
 where   and   have a common divisor that   does not have. If this is 
the  case,  then  we  can  rewrite  the  congruence  as    where  ,  , 
and   are constants and   is not a divisor of  . We will rewrite this with plain algebra as 
as  we  did  before  to  get 
.  However,  this 
is  a  contradiction  because  we  defined    and    such  that    is not a divisor of  . Thus, if 
the  coefficient  of  the  variable  and  the  modulo  share  a  common  divisor  that  the other 
number in our congruence doesn't, the congruence has no solution. 

Next,  we will look at   where   and   are relatively prime. In this case, 


we  could  try  writing  out  the  equations  in  pure  algebraic form, but no solution seems to 
pop  out  to  us  as  it  did previously when we put the equation in that form (try doing this 
and  thinking  about  it  yourself).  Let's  try  to  take  a step back and try to think about what 
we  are  really  trying  to  accomplish  with  this  "division"  we  want  to  define.  We  want  to 
find  a  way  to  make  the  coefficient  of  our  variable 1 so that we know what values of our 
variable  make  our  equation  hold  true.  We  already  know  that  multiplication  works  like 
normal  in  modular  arithmetic,  so  we  want  to  think  if  there  is  some  value  that  we  can 
multiply  both  sides  by  to make the desired coefficient congruent to 1. As you will prove 
in  the  exercises,  it  turns  out  that  when  the  coefficient  of  the  variable  is  relatively  prime 
to  the  modulus,  there  is  always  such  a  value.  For  an  integer  ,  the  number    such  that 
  is  called  the  inverse  of    in    and  we  write    while 
working  in  .  You  will  also  find an algorithmic method to obtain any number's 
inverse  in  proving  the  existence  of  the  inverse  in  the  exercise.  This  method  will  be 

89
expanded  upon  in  the  Diophantine  Equations  lesson.  However,  for  the  AMC  10,  the 
best method of finding inverses is usually just to guess.  

Note  that  taking  an  nth  root  is  similar  to  division  in  that  there  are  often  multiple 
solutions, however there is no nice way to find the nth roots other than guess and check. 

90
Exercises 

4.2.1)  Prove  that  there  exists  an  inverse  of    in    where    by 
showing  an  algorithm  to  find  the  inverse  (Hint:  what  does  gcd(m,n)=1  remind  you  of? 
How  can  you  rephrase  the  inverse  of  a  number  using  a  purely  algebraic  form  to  relate 
it?). 

4.2.2) Find the solutions to . 

 
 
 
 
 
 

91
4.3 Modular Arithmetic Part II 
So  far,  we  have  introduced  the  basics  of  modular  arithmetic  as  far as simple operations 
and  solving  1  variable  linear  equations  goes.  We  will  now  look  at  how  modular 
arithmetic can be used in harder problems which have a less obvious use or solution. 

Sample  Problem  1.  If  a  class  is  arranged  into  rows  of  10  people, 
there  are  3  people  left  over. If the same class is arranged into rows 
of  13  people,  there  are  8  people  left  over.  What  is  the  minimum 
number of people that could be in the class? 

For  this  problem,  you  could  solve  it  fairly  easily  by  guessing  and  checking.  However, 
we  will instead find a way to use modular arithmetic to systematically find a solution so 
that  when  working  with  more  constraints  (more  than  just  the  2  given)  and  guessing 
becomes  harder,  you  will  have  a way to solve the problem. In addition, it would be nice 
if  we  could  find  all  of  the  solutions.  To  start,  we  will  let  the  number  of  people  in  the 
class  be  .  Thus,  writing  the  information  we  have  using  modular  arithmetic,  we  have 
  and  . It doesn't immediately appear obvious what to 
do  here,  so  let's  think  about  what  ideas  we  may have. Let's think about what we would 
do  if  we  wanted  to  guess  and  check  to  find  the  solution  to  this  problem.  We  would 
probably start with 3 and check if it satisfies the   condition (or start with 8 and 
check  the    condition,  it  doesn‘t  really  matter).  Because  this  doesn't  work,  we 
would  repeatedly  add  10  to  the  3  to  keep  the    condition  satisfied  and 
continually  check  if  the  number  we  get satisfies the   condition until we found 
a  number  that  did.  In  other  words,  we  would  have  tested  numbers  in  the  form  of 
  to see if they are congruent to  . Well, we already know how to 
solve  linear  congruences,  so  why  don't  we  just  plug  in  the  restriction   into 
our  ?  We  can,  and  it  will  get  us  exactly  to  where  we  wanted  doing 
pretty  much  the  same  thing  as  we  did  when  guessing  and  checking  just  without  the 
guessing  and  checking. To finish of this problem, we plug our new expression in for a to 
get  .  We  then  subtract  3  from  both  sides  to  get 
,  and  because  10  and 13 are relatively prime we can find the inverse 
of  10  in    by  guessing  if  the  numbers  are  small  or  using  the  Euclidean 

92
Algorithm  (usually  better  for  big  numbers)  to  get 
.  Now,  be  careful  not  to  just  assume  the 
smallest  value  we  could  get  is  7,  because  that  would  be  wrong.  Remember  that  we  are 
trying  to  minimize    not  .  From  here,  we  can  write    from  our  modular 
equation,  and  plugging  this  into  ,  we  get  ,  or  writing  this 
in  modular  form,  we  get  that  our  solutions  are integers that satisfy   
(note  that    makes  sense  because  it  doesn't  change  the  value  of  a  number  in 
or to add 130). Thus, our answer is 73.  

By  looking  at  the process that we just discovered we would use to solve a system of two 


linear  congruences,  we  can  think  about  what we can conclude from trying to generalize 
it.  Using  our  knowledge  of  inverses  of  a  number  in  a  modulo  existing if and only if the 
inter  is  relatively  prime  to  the  modulo,  you  should  be  able  to  see  that  by  using  our 
previous  process  we  can  find  a  solution  to  any  system of 2 linear congruences as long 
as  the  moduli  of  the  two  congruence  are  relativity  primed.  In  addition,  the  solution 
will  be  with  a  modulus  equal  to  the  product  of  the  moduli  of  the  two  congruences. 
This fact (not the solution process) is called the Chinese Remainder Theorem. 

Sample  Problem  2.  How  would  we  solve  a  system  of  three  linear 
congruences  using  the  process  we  just  made?  How  about  a 
system  of  more  than  three  linear  congruences?  How  does  the 
Chinese  Remainder  Theorem  work  with  more  than  2 
congruences? 

Thinking  about  how  we  solve  a  system  of  more  than  two  linear  equations,  we  can 
conclude  that  we  can  just  solve  the  congruences  by  taking  2  at  a  time.  In  other  words, 
we  take  any  two  congruences,  solve  them  to  find  one  congruence  that  represents  the 
solutions.  We  then take the new congruences and a another congruence, solve those two 
congruences,  and  so  on  until  we  have  only  one  congruence  left  that  satisfies  all  of  our 
original  ones.  Thus,  we  can  fairly  easily  generalize the Chinese remainder therefrom by 
applying  it  to  each  system  of  two congruences that we work through to see that we will 
get  a  solution  if  the  moduli  of  all  of  the  congruences  are  relatively  prime,  and  in  that 
case  the  modulus  of  the  solution  will  be  the  product  of  all  of  the  moduli  of  our 
individual  congruences.  Now,  solving  all  of  these  congruences  using  this  method may 

93
seem  very  tedious,  and  it  is.  Thus, you won't have to solve any systems with a bunch of 
congruences on the AMC 10 (or else you're probably doing the problem wrong).   

Sample  Problem  3.  Find  the  units  digit  of  .  How  can  you 
relate this to modular arithmetic. 

You  probably  know  how  to  solve  this  problem  without  modular arithmetic. You would 
probably  start  by  writing  out  the  sequences  of  the  units  digits  starting  with   to get 3, 
9,  7,  1.  Thus,you  would  probably  think  that  we  can  keep  on  subtracting  fours  from  the 
exponent  (dividing  by  ,  which  has  a  units  digit  of  1)  to  get  that    has  the  same 
units  digit  ,  so  our  answer  is  7  (we  could  also  see  this  as  saying  that  123  has  a 
remainder of 3 when divided by 4). 

Next,  we  will  relate  this  to  modular  arithmetic.  Thinking  how  to  express  a  units  digit, 
we  realize  that  the  units  digit  of  a  number  is  equivalent  to  that  number  . 
Thinking  about  what  we  doing  in  terms  of  modular  arithmetic  again,  we  see  that  we 
found    as the lowest exponent of 3 that is congruent to 1 in 
,  so  thus  we  factored  out    from    repeatedly.  Now,  this  may  not  initially,  but  we 
can  generalize  this  beyond  the  units  digit  of  a  number.  We  call  the  smallest  number   
such  that    the  order  of  n  .  The  concept  of  the  order  of  a 
number  in  a  mod  will  pop  up  again  in  higher level number theory that is not necessary 
for the AMC 10, but we will give a brief introduction to these ideas in the exercises.  

 
 
 
 
 
 
 
 
 
 
 

94
Exercises 
4.3.1)  Show  that,  for  integers  x,  y,  and  z  such  that  x  is  relatively  prime  to  m  and 
, . 

4.3.2)  Use  the  previous  statement  to  come  up  with  an  alternate  proof  for  the  Chinese 
Remainder Theorem. 

4.3.3)  We  call  the  number of integers less than   that are relatively prime    . Prove 


that for relatively prime to . 

95
Solutions  

1.1.1)  First  of  all,  note  because  the  range  is  1,  the  median  must either be just the middle 
term,  or  the  two  terms  that  average  to  it  must  both  be 8. Thus, we know there is an 8 in 
the  set.  However,  because  the  average  is  greater  than  8,  we  also  need  numbers  greater 
than  8,  which  can  only  be  9s.  We  will  then  break  this  up into cases based on if n is even 
or odd.  
 
First,  if  n  is  even,  we  know  that  we have two 8s in the middle and can add and 8 and 9 
pair  by  putting  one  on  either  side  of  the  two  middle  8s  when  in  non-decreasing  order. 
Thus,  we always have two more 8s than 9s, so our average is  . Setting this equal 
to  8.3,  we  have  .  This 
works because we assumed a is even, and 10 is indeed even. 
 
Second,  if  n  is  odd,  we  can  use  similar  reasoning  to  get  . Solving, we get a 
a=5. This is indeed odd, so thus we get a= 5, 10 as our solutions.  
 
Notice  that  if  you  were  to  try  solving the problem with an average of 8.5 you would get 
no  solutions.  The  same  is  true  for  any  average  below  8.  Try  to  come  up  with  a 
non-algebraic,  intuitive  explanation  for  why  the  average  must  be  between  8  and  8.5, 
including  8  and  excluding  8.5.  Also,  note  that  not  every  value  in  this  interval  can  be 
formed (can you find an intuitive explanation for this). 

1.1.2)  First,  note  that  the  sum  of  the  elements  in  the  set  is  ,  where  a  is  the  average 
value  of  the  elements  in  the  set.  Thus,  when  we  remove  an  element  of value v, the sum 
of  the  remaining  elements  is  reduced  to  ,  and  the  average  becomes  .  Try 
letting to see what the average would become in terms of c. 

1.2.1)  Note  that  the    roots  of  unity  are  the  solutions  to  ,  or  .  By 
Vieta's  Formulas, the sum of the roots of this polynomial is equal to the coefficient of the 
  term  divided  by  the  coefficient  of  .  However,  because  ,  the    term  has  a 
coefficient of and has a coefficient of 1. Thus, the desired sum is .  

96
1.2.2)  Note  that,  by  Vieta's  Formulas,  the  product  of  all  of  possible  values  of    is 
negative  the  value  of  the  constant  term  divided  by  the  coefficient  of  the    term  in  the 
second equation. 

First  of  all,  let's  try  to  find  the  constant  term.  By  Vieta's  Formulas  on  the  first  equation, 
we  get  .  Next,  note  that 
.  Thus,  the  constant  term  is 

Next,  we  will  find  the    term's  coefficient.  Notice  that 


  Thus,  the    term's  coefficient  is 

Therefore, the final answer is  

1.3.1)  We  can  simply  observe  that 

.  However,  when 
we  expand  out  ,  we  see  that  it  telescopes 
nicely  because 

,  

as desired. 

1.4.1)  We  will  simply  turn  the  words  in  the  problems  into equations and then solve. We 
will  let  the  number  of  cups he has be  , the number of ounces of lemonade he had be  , 
and  the  number  of  cups  his  cooler  be  .  From  our  first  given  we  have  that  . 
Second,  we  have  that  .  Third,  we  have  that  .  By  substituting  the  first 
equation  into  the  second  for    and  multiplying  by  2,  we  get 
  Thus,  using  the  last  equation,  we  get    =     
Therefore, our answer is 4. 

1.4.2)  We  will  again  start  by  defining  variables.  Let  Mary's  rate  for  the  first  half  of  the 
track  be    mph  and  the  distance  of  the  total  track  be    miles.  We  have the the first half 

97
of  the  track  takes  3  minutes  or  ,  so  .  Thus,  .  Next,  we  know  she 
decreased  her  rate  by  1  mph  and traveled the second half of the track in 4 minutes or   
hours.  Thus,  we  have    and  .  Equation  \our  two  expressions  for  , 
we get . Therefore, we finally achieve miles. 

2.1.1)  We  will  count  this  quantity  by  taking  the  total  number  of  arrangements  and 
subtracting  the  arrangements  with  two  blue  beads  in  a row. There are a total of   
arrangements  and  a  total  of  4  ways  to  arrange  the  beads  with  the  2  blue  beads  next  to 
each other. Thus, there are 6 arrangements fitting the desired constraints. 

2.2.1)  We  will  solve this problem just as we solved the previous ones. If we add together 


the  number  of  students  in  each  group  of  3  teams  we  will  have  counted  the  students  in 
all  4  teams    times,  so  we  need  to  subtract  the  number of students that are in all 4 
teams  3  times.  Thus,  we  get  that  the  total  number  of  people  in  at  least  3  clubs  to  be 

2.2.2)  We  will  use  complementary  counting.  First,  we  will  count  the  numbers  of 
multiples  of  3  or  8  below  101.  We  can  easily  see  that  there  are  33  multiples  of  3,  12 
multiples  of  8,  and  4  multiples of 3 and 8. Thus, there are 33+12-4=41 multiples of 3 or 8 
below 101 and 100-41=59 positive integers that are not multiples of 3 or 8 below 101. 

2.3.1)  We  will  use  complementary  counting  to  reduce  the  amount  of  casework  that  we 
have  to  do.  If  we  don't  have  at  least  2  heads come up, then that means that either 0 or 1 
head  comes  up.  The  probability  that  exactly  0  heads come up is  . The probability that 
exactly  1  head  will  come  up  is    because  any  of  the  coins  could  come  up heads. Thus, 
the probability that at least 2 heads come up is 1- - = . 

2.3.2)  We  will  do casework on the number of 4s. If we have 0 4s, we can have 0-6  2s and 


fill  the  rest  with  1s  for  a  total  of  7  possibilities.  Similarly,  for  1,  2,  and  3 4s we have 5, 3, 
and 1 possibilities respectively. Thus, there are a total of 1+3+5+7=16 total ways. 

2.4.1)  By  the  Binomial  Theorem,  when  we  expand    we  merely  get  the  sum 
of the numbers in the nth row of pascal’s triangle. 

98
2.4.2)  There  are  several  ways  to  prove  this  identity,  we  will  just  show  one.  Using 
pascal’s  triangle,  we  know  that  each  number  is  equal  to  the  sum  of  the  two  numbers 
directly  above  it.  In  other  words,  we  get  that  the  sum  of  the  kth  and  k+1th  numbers  in 
the  nth  row  is  equal  to  the  kth  number  in  the  n+1th  row.  However,  using  the  binomial 
coefficient  definition  of  Pascal's  Triangle,  we  get  can  restate  the  above  to  get 
, which is exactly what we wanted. 
 
This identity that we just proved is called Pascal's Identity. 

2.4.3)  There  are  again  several  ways  to  prove  this.  We  will  only  show  an  algebraic proof 
here,  but  we  encourage  you  to  try  to  find  another  proof  using  Pascal's  Triangle  as  we 
did in the above solution.  
 
We  will  perform  the  following  steps  using  Pascal's  Identity  (exercise  2)  and  basic 
properties of binomial coefficients: 

 
And thus, we are done. 

3.1.1)  From  the  equation  A=rs,  we  have r=A/s. Using similar triangles, we can find that 


the  triangle  formed  by  connecting  the  larger  triangle's  midpoints  has  1/4  the  area  and 
1/2  the  semiperimeter  (each  side  of  the  triangle  can  be  easily  count  parallel  to  the  side 
of  the  triangle  that  wasn't  used  in  forming  that  side,  and  from  there  we  can  easily  find 

99
the  inner  triangle  similar  to  the  outside  one  with  a  side ratio of 1/2). Thus, the inradius 
of  the  smaller  triangle  is  divided  by  4 and multiples by 2, leaving a ratio of the inradius 
of the outer triangle to the inradius of the inner triangle to be 2 to 1. 

3.2.1)  Note  that    and  .  Thus, 


as desired. 

3.2.2)  We  will  simply  note  that  triangle  DFC  is  similar  to  triangle  EGC  because  angles 
CEG  and  CDF  because  they  intersect  the  same  arc.  Thus,  we  get 

 
OR 
 
Similarly  to  how  we  proved the last case of the Power of a Point theorem, both   
and are equal to the power of point C with respect to circle O. 

3.3.1)  We  know  that  the  centroid  is  the  intersection  of  the  medians.  Thus,  we  simply 
need  to  find  the  formulas  for  the  medians.  The  midpoint  of  AB  is  (2,3),  so  we  get  that 
the  median  to  C  has  the  equation  y=3x-3.  The  midpoint  of  BC  is  (3,5),  so  the  equation 

100
for the median to A is y=3x/2+1/2. These two medians intersect at .  
 
Try  to generalize this for a triangle with vertices  ,  , and   to see if you 
can find a general formula. 

3.3.2)  To  find  E,  we  will  just find the intersection between line BD and circle B though A 


keeping  in  mind  that  E  has  an x value between 0 and 1. note that the circle has equation 
.  Line  BD  has  equation  y=-x+1.  Thus,  they  intersect  at  . 
However,  because  we  know  x  is less than 1, we see that E is at  . Thus, the area 

of triangle AEB is . 

3.4.1)  Note  that,  by  symmetry,  the  center  of  the  cube  is  also  the  center  of  the  sphere. 
Thus,  long  diagonal  of  the  cube  has  length  equal  to  the  diameter  of  the  cube,  or  . 
Next,  we  will  find  the  length  of  the  long  diagonal  of  a  cube  in  terms  of  the  cube's  side 
length in order to find the cube's side length. 

Note  that  the  long  diagonal  is  the  hypotenuse  of  a  right  triangle  with  legs  equal  to  the 
diagonal  of  one  face  and  one  of  the  sides.  Thus,  the  square  of  the  length  of  the  long 
diagonal  is  equal  to  to  the  sum  of  the  square  of  the length of one side and the square of 
the  length  of  the  diagonal  of  one  face.  However,  by  the  Pythagorean  Theorem,  we  also 
get  that  the  square  of  the  length  of  the  diagonal  of  one  facts  is  equal  to  the  sum  of  the 
square of the length of two sides of the cube. 

Thus,  the  square  of  the  length  of the diagonal of the cube is equal to thrice the square of 


the  length  of  one  side.  In  other  words,  if  d is the long diagonal's length and s is the side 
length,  we  get  .  Thus,  .  Thus,  the  cube  has  a 
volume of 8. 

3.4.2)  Note  that  the  shape  formed  by  connected  these  centers  is  just  a  regular 
octahedron.  To  find  the  volume  of  the octahedron, we will split it up into two pyramids 
sharing  a  base  that  is  on  the  plane  parallel  to  the  base  of  the  cube  going  through  the 
midpoint  of  each  side  perpendicular  to  the  base.  We  can  easily  see that this base has an 
101
area  of  half  that  of  the  top  face  of  the  cube,  or  an  area  of  1/2.  Because  we  divided  the 
octahedron  into  2,  we  can  easily  see  that  the  height  of  each  pyramid  is  also  1/2.  Thus, 

the volume of the tetrahedron is equal to .  

3.5.1)  We  just about proved this problem already in sample problem two. All we need to 


do  for  this,  is  rotate  and  translate  the  smaller  of  the  two  similar  triangles  so  that one of 
its  angles  coincides  with  an  angle  of  the  larger  triangle,  such  that  the  two  sides around 
the  angle  in  each  triangle  are  collinear.  The  two  triangles  will  then  look  like  one  of  the 
three  diagrams  in  sample  problem  2,  and  we  can  use  the  Pythagorean  theorem  as 
described in sample problem 2 to finish the problem off. 

3.5.2)  Note  that  triangle  EFB  is  similar  to  triangle  CEB  because  angle  EFB  equals  angle 
CEB  ancf/d  angle  B  equals  itself.  Thus,  we  get  that  EF/FB=CE/EB.  Similarly,  we  see 
that  triangle  EFC  is  similar  to  triangle  CEB,  meaning  that  CF/EF=CE/EB.  Thus,  we 
have EF/FB=CF/EF or, equivalently, . 

3.6.1)  First  of  all,  we  don't  need  to  worry  about  the  signs  of  the  sines  because  we  are 
squaring  them.  Next,  this theorem reminds us of the Pythagorean Theorem. In addition, 
we  saw  in  the  first  diagram  that  the  sine and cosine represent lengths in a right triangle 
in  the  unit  circle.  However,  through  this  observation  we  are  done!  The  hypotenuse  of 
the  right  triangle  seen  is  the  first  diagram  is  the  radius of the unit circle, and thus it has 
length  of  1.  Thus,  for  any  angle,  we  will  get  the  desired  equality  simply  by  using  the 
Pythagorean Theorem and the unit circle definitions for sine and cosine. 

3.6.2)  See  the  values  that  we  saw  earlier  and  angles  that  are  related  to  ,  we  think 
equilateral  triangles.  We  will  draw  the  equilateral  triangle  with side length 2. However, 
this  doesn't  get  us  a    angle  or  an  altitude.  Thus,  as  we know that the altitude from a 
vertex  of  an  equilateral  triangle  is  also  an  angle  bisector,  we  draw  an  altitude.  From 
102
there,  we  have  both  of  the  desired  angles,  so  we  just  need  to  find  CD.  We  can  do  this 
easily  using  the  Pythagorean  Theorem  to  get  .  Thus,  we  get 
that and . 

4.1.1)  We  will  prove  this  by  showing  that  applying  the  Euclidean Algorithm to   
is  bound  to  eventually  yield  .  To  start,  we  already  know  that  if  we  ever  get 
down  to  ,  then  g=d  because  we  already  proved  that  the  Euclidean  Algorithm 
will  yield  the  correct  greatest  common  divisor.  Next,  if  at  any  point    is  reduced 

103
to  ,  then  we  apply  the  Euclidean  Algorithm  again  to  get  to  ,  meaning 
r=d, and we have achieved the desired outcome. 

We  will  then  assume  for  the  sake  of  contradiction  that we can go without ever reaching 
two  equal  numbers.  Thus,  if  repeatedly  applying  the  Euclidean  Algorithm  doesn't 
reduce  us  to  a  case  with  two  equal  numbers,  one  number  will  always  be  greater  than 
the  other.  Thus,  if  one  number  is  always  greater,  we  can  continually  take  the  lesser 
number  and  subtract  it  from  the  greater. This process will clearly never make one of the 
two  numbers  negative  because  or non-integral because we are taking one integer minus 
a  smaller  integer.  However,  both  numbers  are  finite,  so  if  we  repeat  this  process  an 
arbitrarily  large  number  of  times  we  will  be  subtract  at  least  one  from  one  of  the 
numbers  each  time,  so  we  will  be  subtracting  an  arbitrarily  large  amount  from  their 
finite  sum.  However,  this  is  a  contradiction  because  this  would  make  at  least  one 
number negative, but we said earlier that they must both be positive. 

Therefore, we will eventually reach , as desired. 

4.1.2)  First,  note  that  13  and  28  are  clearly  relatively  prime.  Thus,  using  the  Euclidean 
Algorithm  will  eventually  get  us  to  .  In  other  words,  we  created  the  1  by 
subtracting  two  numbers,  both  of  which  were  formed  by  subtracting  two  larger 
numbers,  and  so  on  until  each  number  was  formed  from  our  original  two  numbers. 
Thus,  we  will  essentially  start  from  the  bottom  of  the  euclidean  algorithm  and  use 
substitutions  to  get  us  back  to  only  using  the  first  two  numbers.  We  will  start  by 
computing and showing each step. 

104
 
Now,  we  could  have  gone  one  step  further,  but  we  only  needed  to  prove  that  we 
reached  a  0  because  it  helped  us  prove  we  would  reach the greatest common divisor as 
one  of  the two numbers as well. This is because now that we have a 1, we have all of the 
substitutions  we  need  (we  aren't  trying  to  find  solutions  to  ). 
Continuing on to do the problem, we just substitute as we described before.  
 
 
 
 
 
Thus,  we  get  x=13,  y=-6  as  a  solution  using  the  euclidean  algorithm.  You'll  see  this 
more  in  the  Diophantine  Equations  lesson  and  you'll  how  this  is  useful  after  the 
Modular Arithmetic lessons. 

4.2.1)  First  of  all,  note  that  finding  an  inverse  is  the  same  as  solving  the  modular 
congruence  ,  or  in  algebra,  .  Recall 
exercise  2  from the Euclidean Algorithm that we can use the Euclidean Algorithm going 
backward  to  express  1  in  the  form  that  gives  solutions  to  this  equation  when    and   
are  relatively  prime  as  given  (exercise  2  was  a  specific  example,  but  can  be  easily 
general).  If  you  don't understand this exactly, we will fully explain it in the Diophantine 
Equations lesson. 

4.2.2)  We  will  start  by  dividing  the  whole congruence by 4 to get  . Next, 


using  the  Euclidean  Algorithm  or  by  guessing  and  checking,  we  get  that  8  has  an 
inverse  of  5  in  .  Thus,  multiply  both  sides  of  the  congruence  by  5,  we  get 

4.3.1)  We  will  start  by  dividing  the  whole congruence by 4 to get  . Next, 


using  the  Euclidean  Algorithm  or  by  guessing  and  checking,  we  get  that  8  has  an 

105
inverse  of  5  in  .  Thus,  multiply  both  sides  of  the  congruence  by  5,  we  get 

4.3.2)  We  wish  to  prove  that  there  is  a  solution  to  the system  ,   
for  m  relatively  prime  to  t.  We  will  rewrite  the  first  condition  as    for  some 
integer  y.  Next,  note  that  using  the  previous  exercise,  we  can  see  that    changes 
value   whenever y changes in value. In other words, as y ranges over all residues 
in  ,  the  value  of    same  twice.  Thus,  we  can  conclude  that the value of   
must  also  range  over  all  residues  .  Thus,  for  some  value of y,   will assume 
the  value  of  ,  thus  showing  that  we  do  indeed  have  one  solution  that  is  less  than 
  (the  greatest  residue    is  t-1).  Thus, any remaining solutions must be the one 
solution  that  we  found  plus  a  multiple  of  (as  the  number  we  add  must  be  a 
multiple of both moduli). 

4.3.3)  First  of  all,  note  that  when  two  numbers  that  are  both  relatively  prime  to  the 
modulus  are  multiplied,  the  result  must  also  be  relatively  prime  to  the  modulus (if this 
isn't  clear  try  writing  it  out  in  non-modular  notation  and  use  a  proof by contradiction). 
Thus,  if  take  the  set  of  integers  relatively  prime  to  and  less  than  modulus  ,multiply 
each  by  a,  and  take  the  remainder  of  each  when  divided  by  ,  we  simply  get  a 
rearrangement  of  the  set  of  numbers  relatively  prime  to  and  less  than  m  (by  exercise 1, 
no  two  distinct  integers  can  produce  results  congruent    when  multiplied  by  a, 
but,  as  we  said  earlier,  each  product  must also produce a residue relatively prime to the 

modulus).  Thus,  we  have   


where  each    is  a  distinct  residue    that  is  relatively  prime  to  m.  However, 
because   is relatively prime to m, it also has an inverse. Thus, multiplying by each side 
by  in  the  inverse  of  each    we  get  .  Note  that  this  isn't 
necessarily  the  order  of  a,  as  there  could  be  a  smaller  power  congruent  to  1,  but  the 
order must be a divisor of (try to prove this). 

106

S-ar putea să vă placă și